You are on page 1of 69

Elderly Care & Fragility  QUESTION 1 OF 12

A 68 year old woman is brought to the Emergency Department by her concerned son.
He describes several months of mild confusion and disinhibition. He has not noted
any change in her mood. She has frequent falls and has become incontinent of urine
over the last month. She has no significant past medical history and does not smoke
or drink alcohol. General physical examination is unremarkable. On neurological
examination you note normal cranial nerves and upper limb neurology however the
lower limbs have an increased tone with upgoing plantars. What is the diagnosis?

Alzheimer’s disease 8%

Benign intracranial hypertension 6%

Lewy body dementia 21%

✓ Normal pressure hydrocephalus 53%

x Vascular dementia 13%

ANSWER

Whilst falls and urinary incontinence can occur in Alzheimer’s disease, these occur late in disease
progression and do not fit with the patient’s young age or mild cognitive impairment.
Lewy body dementia has hallucinations as a prominent feature (which are not mentioned) and
commonly coexists with parkinsonian movement symptoms. This woman seems to have pyramidal
signs in the legs and no arm symptoms.
Vascular dementia occurs with multiple strokes or transient ischaemic attacks (TIAs), which this
woman does not have a history of, or risk factors for.
Benign intracranial hypertension does not commonly cause dementia, and was mainly included to
confuse, with its mention of intracranial pressure. It presents with headaches, visual disturbance
(papilloedema, which would be absent with normal pressure) and sometimes VI cranial nerve palsies.
Normal pressure hydrocephalus commonly presents with the triad of dementia, gait disturbance and
urinary incontinence.
Dementia LAST UPDATED: 12TH
FEBRUARY 2021
ELDERLY CARE & FRAGILITY
 Bookmark

Dementia is a typically progressive clinical syndrome of deteriorating mental function significant


enough to interfere with activities of daily living (ADLs). It affects cognitive domains (such as memory,
thinking, language, orientation and judgement) and social behaviour (such as emotional control and
motivation).

Causes
Alzheimer’s disease (50 – 75% of cases)
Atrophy of the cerebral cortex
Formation of amyloid plaques and neurofibrillary tangles
Reduced neuronal acetylcholine production
Vascular dementia (up to 20% of cases)
Reduced blood supply to the brain
Dementia with Lewy bodies (DLB) (10 – 15% of cases)
Cortical and subcortical Lewy bodies (abnormal deposits of protein inside nerve cells)
Frontotemporal dementia (FTB) (2% of cases)
Progressive degeneration of the frontal and/or temporal lobe
Rarer causes of dementia:
Parkinson’s disease dementia
Progressive supranuclear palsy
Huntington’s disease
Prion disease (such as Creutzfeldt-Jakob disease [CJD])
Normal pressure hydrocephalus (NPH)
Chronic subdural haematoma
Benign tumours
Metabolic and endocrine disorders (such as chronic hypocalcaemia and recurrent
hypoglycaemia)
Vitamin deficiencies (such as B12 and thiamine deficiency)
Infections (such as HIV infection and syphilis)

Risk factors
Age - strongest risk factor for dementia
Mild cognitive impairment
Learning difficulties
Genetics
Cardiovascular disease risk factors (such as diabetes, smoking, hypercholesterolemia, and
hypertension)
Parkinson's disease
Stroke
Depression
Heavy alcohol consumption
Low educational attainment
Low social engagement and support

Clinical features
Dementia can be difficult to identify as it usually has an insidious onset and non-specific signs and
symptoms, which vary from person to person. People with early dementia may deny symptoms or
accommodate to cognitive change and functional ability.
Suspect dementia if any of the following are reported by the person and/or their family/carer:
Cognitive impairment, including:
Memory problems
Receptive or expressive dysphasia
Difficulty in carrying out coordinated movements such as dressing
Disorientation and unawareness of the time and place
Impairment of executive function, such as difficulties with planning and problem solving
Behavioural and psychological symptoms of dementia (BPSD) tend to fluctuate, may last for 6
months or more and include:
Psychosis
Agitation and emotional lability
Depression and anxiety
Withdrawal or apathy
Disinhibition
Motor disturbance
Sleep cycle disturbance or insomnia
Tendency to repeat phrases or questions
Difficulties with activities of daily living (ADLs):
In the early stages of dementia this may lead to neglect of household tasks, nutrition
(causing weight loss), personal hygiene, and grooming. People with dementia who are in
employment may find that they are increasingly making mistakes at work.
In the later stages, basic ADLs such as dressing, eating, and walking become affected.
Specific features of subtypes of dementia:
For Alzheimer’s disease:
Early impairment of episodic memory — this may include memory loss for recent events,
repeated questioning, and difficulty learning new information.
For vascular dementia:
Stepwise increases in the severity of symptoms — subcortical ischaemic vascular dementia
may present insidiously with gait and attention problems and changes in personality.
Focal neurological signs (such as hemiparesis or visual field defects) may be present.
For dementia with Lewy bodies:
Repeated falls, syncope or transient loss of consciousness, severe sensitivity to
antipsychotics, delusions, and hallucinations may be present. Memory impairment may not
be apparent in early stages. Parkinsonian motor features (such as shuffling gait, rigidity,
bradykinesia, and loss of spontaneous movement) and autonomic dysfunction (such as
postural hypotension, difficulty in swallowing, and incontinence or constipation) may be
present.
For frontotemporal dementia (FTD):
Personality change and behavioural disturbance (such as apathy or social/sexual
disinhibition) may develop insidiously. Other cognitive functions (such as memory and
perception) may be relatively preserved.

Differential diagnosis
Conditions that can present with similar symptoms to dementia include:
Normal-age related memory changes - Normal ageing is associated with a mild decline in
cognitive function, and memory lapses are common, especially during times of physical illness or
stress.
Mild cognitive impairment (MCI) - MCI differs from dementia in that symptoms do not fulfil the
diagnostic criteria for dementia, for example only one cognitive domain may be affected or
activities of daily life may not be significantly affected. Of people with MCI, 50% will later develop
dementia.
Depression - Symptoms of depression include low mood, loss of interest, anhedonia, and self-
neglect which can be similar to those of dementia.In older people, features of depression may be
less obvious, with somatic symptoms (such as reduced appetite, fatigue, and insomnia) are more
common.
Delirium - Delirium is an acute, fluctuating syndrome of disturbed consciousness, attention,
cognition, and perception. It is a common condition in the differential diagnosis for dementia.
People with cognitive impairment are at increased risk of delirium, and the two conditions often
coexist.
Vitamin deficiency - Thiamine deficiency can lead to Wernicke encephalopathy and Korsakoff
psychosis. Symptoms include confusion, memory loss, problems with learning new information
and gait disturbances. Vitamin B12 deficiency can lead to ataxia, psychiatric abnormalities,
memory loss, and gait disturbance.
Hypothyroidism - Symptoms of hypothyroidism can include low mood, and impaired
concentration and memory.
Adverse drug effects - Many drugs, including benzodiazepines, analgesics (such as opioids,
naproxen, and ibuprofen), anticholinergics, antidepressants (such as tricyclics), antipsychotics
(such as haloperidol), anticonvulsants (especially older preparations, such as phenytoin and
phenobarbital), and corticosteroids can affect cognition.
Normal pressure hydrocephalus - Normal pressure hydrocephalus can present with symptoms of
early cognitive impairment, urinary incontinence, and gait disorder. In NPH, cerebrospinal fluid
(CSF) flow has been impeded but compensatory mechanisms have prevented raised pressure.
The lateral ventricles are prominently dilated, and these ventricles exert local pressure on certain
brain areas to give the classical clinical picture. Pressure on the frontal lobes gives the dementia
and pressure on the medial side of the motor cortex, and the pyramidal tract fibres, cause
incontinence and pyramidal leg weakness. Diagnosis is by lumbar puncture (to demonstrate a
normal CSF opening pressure) followed by head computed tomography (CT)/magnetic resonance
imaging (MRI) (showing enlarged ventricles). Treatment is with ventriculoperitoneal shunting.
Sensory deficits - Problems with vision and hearing can contribute significantly to an apparent
decline in cognitive ability.

Investigations
Patient's should be assessed using a standardised cognitive assessment tool.
To help identify reversible causes of dementia and exclude other causes of symptoms initial
investigations include:
In most cases:
Full blood count
Erythrocyte sedimentation rate (ESR)
Urea and electrolytes
Calcium
HbA1c
Liver function tests
Thyroid function tests
Serum B12 and folate levels
If clinically indicated:
A midstream urine, for example when delirium is a possibility
Chest X-ray, electrocardiogram (ECG), syphilis serology, and HIV testing
Patients with suspected dementia should be referred to a specialist memory assessment service
for further specialist assessment and management.
Specialist investigations for suspected dementia:
Should include structural imaging (MRI or CT scan) to: exclude non-dementia cerebral
pathology such as normal pressure hydrocephalus and to identify dementia subtype.
May also include:
HMPAO SPECT and FDG PET as second line investigations if diagnosis or subtype is
unclear
(FP-CIT) SPECT where a potential diagnosis of dementia with Lewy bodies (DLB) is in
doubt
Cerebrospinal fluid examination to exclude inflammatory, infective, or malignant
causes of dementia, in cases where dementia is rapidly progressive, the presentation
is unusual, or the person is younger than 55 years of age
Electroencephalography if a diagnosis of delirium, frontotemporal dementia, or
Creutzfeldt–Jakob disease is suspected, or in the assessment of associated seizure
disorder in people with dementia
Brain biopsy in highly selected people whose dementia is thought to be due to a
potentially reversible condition (such as cerebral vasculitis) that cannot be
diagnosed in another way
Management
Specialist non-pharmacological interventions for cognitive symptoms of dementia:
Structured group cognitive stimulation programs
Reminiscence therapy with discussion of past experiences
Tools such as life histories, shared memories, and familiar objects from the past
Specialist drug treatments for cognitive symptoms of dementia:
Acetylcholinesterase (AChE) inhibitors (donepezil, galantamine, and rivastigmine) - These
drugs can be used for mild to moderate Alzheimer's disease.
Memantine (a N-methyl-D-aspartic acid receptor antagonist) - Memantine is an option for
managing Alzheimer's disease for people with moderate Alzheimer's disease who are
intolerant of, or have a contraindication to, AChE inhibitors or in severe Alzheimer's disease.
Specialist drug treatment for behavioural and psychological symptoms of dementia (BPSD) or
non-cognitive symptoms of dementia and challenging behaviour:
No antipsychotics (except risperidone in very specific circumstances) are licensed for
BPSD in the UK. Specialists may prescribe an antipsychotic drug off-label in certain
circumstances (e.g. if they are severely distressed or there is an immediate risk of harm to
the person or others) following a thorough clinical assessment and discussion (including
the risks and benefits) with the carer and (if possible) the person with dementia.
Antipsychotics have potentially serious adverse effects, including increased risk of stroke
and mortality, Parkinsonism, and cognitive impairment. They should be used with caution,
at low dose, and for the shortest time possible.
In specific situations, specialists may offer AChE inhibitors for BPSD if non-
pharmacological methods and anti-psychotic drugs are inappropriate or ineffective.
Elderly Care & Fragility  QUESTION 2 OF 12

A 68 year old woman presents to the Emergency Department following a fall. She is
complaining of right hip pain and is unable to weight bear. Her x-ray shows a
fractured neck of femur and you consider if she has underlying osteoporosis.
Regarding osteoporosis, which of the following statements is CORRECT?

Osteoporosis is a disorder of defective mineralisation of bone matrix. 7%

Osteoporosis is usually diagnosed using a full skeletal survey. 1%

The proximal humerus is the most common site of fracture after minimal trauma in 2%
osteoporosis.

✓ Osteoporosis occurs most commonly in postmenopausal women. 86%

Osteoporosis occurs when bone density is >1 standard deviation below normal peak bone 3%
x mass.

ANSWER
Osteoporosis is a disease characterised by reduced bone mass and increased bone fragility resulting in an
increased risk of fracture after minimal trauma (particularly of the hip, spine and distal radius). It is very
common in postmenopausal women. Bone densitometry, measured by dual energy X-ray absorptiometry
(DXA) scan, is the mainstay of diagnosis. Osteoporosis, defined according the the T score, occurs when
bone density is >2.5 standard deviations below normal peak bone mass.

Osteoporosis and Fragility Fractures LAST UPDATED: 5TH MARCH


2021
ELDERLY CARE & FRAGILITY
 Bookmark

Osteoporosis is a disease characterised by low bone mass and structural deterioration of bone tissue,
with a consequent increase in bone fragility and susceptibility to fracture. Osteoporosis is the end
result of an imbalance in the normal process of bone remodelling by osteoclasts and osteoblasts. It is
asymptomatic and often remains undiagnosed until a fragility fracture occurs.

Definitions
Osteoporosis is defined by the World Health Organization as a bone mineral density (BMD) of 2.5
standard deviations below the mean peak mass (average of young healthy adults) as measured
by dual-energy X-ray absorptiometry (DXA) applied to the femoral neck and reported as a T-
score.
An osteoporotic fracture is a fragility fracture occurring as a consequence of osteoporosis.
Osteoporotic fractures characteristically occur in the wrist, spine and hip, but can also occur in
the arm, pelvis, ribs and other bones.
A fragility fracture is defined as a fracture following a fall from standing height or less, although
vertebral fractures may occur spontaneously, or as a result of routine activities such as bending
or lifting.

Risk factors
The risk of getting an osteoporotic fracture depends on the person's risk of falls, their bone strength
(determined by bone mineral density), and other risk factors.
Risk factors for having a fragility fracture:
Risks factors affecting bone strength that reduce BMD:
Endocrine disease e.g. diabetes mellitus, hyperthyroidism, hyperparathyroidism
Malabsorptive disease e.g. IBD, coeliac disease, chronic pancreatitis
Chronic kidney disease
Chronic liver disease
COPD
Menopause
Immobility
BMI < 18.5 kg/m2
Risks factors affecting bone strength that do not reduce BMD:
Age
Oral corticosteroids
Smoking
Alcohol
Previous fragility fracture
Rheumatological disease e.g. rheumatoid arthritis
Parental history of hip fracture
Risk factors affecting bone strength with unestablished mechanisms:
Selective serotonin reuptake inhibitors (SSRIs)
Proton pump inhibitors (PPI)
Anticonvulsant drugs e.g. carbamazepine
Risk factors for falls:
Impaired vision
Neuromuscular weakness and incoordination
Cognitive impairment
Use of alcohol and sedative drugs

Assessment for fragility fracture risk


Exclude non-osteoporotic causes for fragility fractures:
Metastatic bone disease (bone pain, history of cancer (especially lung, thyroid, prostate,
kidney, or breast cancer), or symptoms of undiagnosed cancer (for example unexplained
general malaise or weight loss)
Multiple myeloma (bone pain, anaemia, recurrent infection, bleeding, hypercalcaemia,
kidney disease)
Osteomalacia (bone pain, muscle pain or proximal muscle weakness)
Paget’s disease (bone pain or deformity)
Aneurysmal bone cyst or hemangioma
Infection e.g. osteomyelitis, Pott's disease
Exclude secondary causes of osteoporosis especially in people with a fragility fracture who are at
low risk, including men of any age, pre-menopausal women, and women with premature
menopause (under 40 years of age)
Consider starting drug treatment in people with vertebral or hip fractures without undertaking
DXA if this is considered inappropriate or impractical
Offer a dual-energy X-ray absorptiometry (DXA) scan to measure bone mineral density (BMD)
without calculating the fragility fracture risk in people:
Over 50 years of age with a history of fragility fracture
Younger than 40 years of age who have a major risk factor for fragility fracture
For all other people with risk factors for osteoporosis, calculate the 10-year fragility fracture risk
prior to arranging a DXA scan to measure BMD; consider using the online risk calculators
QFracture® (preferred) or FRAX®, which predict the absolute risk of hip fracture and major
osteoporotic fractures (spine, wrist, hip, or shoulder) over 10 years
High risk: For people whose fracture risk is above the recommended threshold, offer a dual-
energy x-ray absorptiometry (DXA) scan, then bone-sparing drug treatment if the T-score
is -2.5 or less; if the T-score is greater than -2.5, modify risk factors where possible, treat
any underlying conditions, and repeat the DXA at an interval appropriate for the person
based on their risk profile, using clinical judgement (but usually within 2 years)
Intermediate risk: For people whose fracture risk is close to the recommended threshold
and who have risk factors that may be underestimated by FRAX®, arrange a DXA scan to
measure their bone mineral density (BMD) and offer drug treatment if the T-score is -2.5 or
less
Low risk: For people whose fracture risk is below the recommended threshold, do not offer
drug treatment, offer lifestyle advice and follow up within 5 years
Assess for vitamin D deficiency and inadequate calcium intake.
People are at risk of vitamin D deficiency if they are aged over 65 years, or are not exposed
to much sunlight (because they are confined indoors for long periods, or because they wear
clothes that cover the whole body)
A calcium intake of at least 1000 mg/day is recommended for people at increased risk of a
fragility fracture
Identify any risk factors for falls

Management
Lifestyle advice
Advise the person to:
Take regular exercise to improve muscle strength
Eat a balanced diet
Stop smoking if needed
Drink alcohol within recommended limits
Bone-sparing treatment
Prescribe a bisphosphonate (alendronate 10 mg once daily or 70 mg once weekly, or
risedronate 5 mg once daily or 35 mg once weekly), if there are no contraindications and
after appropriate counselling, to postmenopausal women and men over 50 years of age
who have been confirmed by dual-energy X-ray absorptiometry (DXA) scan to have
osteoporosis (bone mineral density [BMD] T-score of -2.5 or less).
Consider prescribing to people who are taking high doses of oral corticosteroids (more than
or equivalent to prednisolone 7.5 mg daily for 3 months or longer).
If an oral bisphosphonate is not tolerated or is contraindicated, consider specialist referral.
Specialist treatment options include zoledronic acid, strontium ranelate, raloxifene,
denosumab, and teriparatide.
Calcium and vitamin D supplements
If the person's calcium intake is adequate (700 mg/day), prescribe 10 micrograms (400
international units) of vitamin D (without calcium) for people not exposed to much sunlight.
If calcium intake is inadequate prescribe 10 micrograms (400 international units) of vitamin
D with at least 1000 mg of calcium daily; prescribe 20 micrograms (800 international units)
of vitamin D with at least 1000 mg of calcium daily for elderly people who are housebound
or living in a nursing home.
Hormone replacement therapy
Consider prescribing hormone replacement therapy (HRT) to women who have a premature
menopause (menopause before 40 years of age) to reduce the risk of fragility fractures and
for the relief of menopausal symptoms.

Vertebral compression fractures


Vertebral compression fractures are the most common type of osteoporotic fracture. They often occur
at the midthoracic (T7 - T8) spine and the thoracolumbar junction (T12 - L1).
Fractures may result in significant back pain, limited physical functioning and activities of daily living,
and can lead to loss of independence, depression, and chronic pain. Osteoporotic vertebral
compression that occurs slowly over time is often asymptomatic. Old or healed fractures may be an
incidental finding on radiographs of the chest or abdomen. In other patients, the presence of vertebral
fractures may become apparent because of height loss or kyphosis. In contrast, acute episodes of
significant vertebral body compression are associated with pain.
In patients who have acute symptomatic vertebral body fracture, there is often no history of preceding
trauma. The typical patient presents with acute back pain after sudden bending, coughing, or
lifting. The pain is usually well localised to the midline spine but often refers in a unilateral or bilateral
pattern into the flank, anterior abdomen, or the posterior superior iliac spine. On physical examination,
the patient may experience pain upon palpation and percussion of the corresponding spinous process
and paravertebral structures.
Acute episodes of pain following a vertebral body fracture usually resolve after four to six weeks, but
pain may persist for longer periods (many months), indicating an unhealed or slowly healing
fracture. The initial management of osteoporotic vertebral compression fractures should include pain
control and activity modification (bed rest is not recommended). The indications for and timing of
vertebral augmentation procedures are controversial. Most patients with vertebral compression
fractures can be treated successfully with conservative management.
Elderly Care & Fragility  QUESTION 3 OF 12

A 78 year old man is brought to the Emergency Department by his wife. She
describes worsening confusion. The nurse at triage feels the patient may have
delirium. Which of the following features is NOT typical of delirium?

Usually develops over hours to days 7%

Behaviour disturbance and personality changes may occur 4%

Symptoms typically fluctuate throughout the day 8%

✓ Lucid intervals usually occur at night 74%

x Typically occurs due to a precipitating factor 6%

ANSWER
Delirium (sometimes called 'acute confusional state') is an acute, fluctuating syndrome of disturbed
consciousness, attention, cognition and perception.

Delirium usually develops acutely over hours to days and behavioural disturbance, personality changes,
and psychotic features may occur. Symptoms typically fluctuate (come and go or increase and decrease in
severity). Lucid intervals usually occur during the day with the worst disturbance at night.

Delirium typically occurs in people with predisposing factors (such as advanced age or multiple
comorbidities) when new precipitating factors (such as some medications or infection) are added.

Delirium LAST UPDATED: 12TH


FEBRUARY 2021
ELDERLY CARE & FRAGILITY
 Bookmark
Delirium (sometimes called 'acute confusional state') is an acute, fluctuating syndrome of disturbed
consciousness, attention, cognition and perception.
Delirium usually develops acutely over hours to days and behavioural disturbance, personality changes,
and psychotic features may occur. Symptoms typically fluctuate (come and go or increase and
decrease in severity). Lucid intervals usually occur during the day with the worst disturbance at night.
Delirium typically occurs in people with predisposing factors (such as advanced age or multiple
comorbidities) when new precipitating factors (such as some medications or infection) are added.

Predisposing factors
Predisposing factors include:
Older age (over 65 years)
Cognitive impairment (such as dementia)
Frailty/multiple comorbidities (such as stroke or heart failure)
Significant injuries such as hip fracture
Functional impairment (for example immobility or the use of physical restraints such as cot sides)
Iatrogenic events (such as bladder catheterisation, polypharmacy, or surgery)
History of, or current, alcohol excess
Sensory impairment (such as visual impairment or hearing loss)
Poor nutrition
Lack of stimulation
Terminal phase of illness

Precipitating factors
Precipitating factors include:
Infection such as urinary tract infection, infected pressure sore, or pneumonia
Metabolic disturbance such as hypoglycaemia, hyperglycaemia, or electrolyte abnormalities
Cardiovascular disorders such as myocardial infarction or heart failure
Respiratory disorders such as pulmonary embolism or exacerbation of chronic obstructive
pulmonary disease
Neurological disorders such as stroke, encephalitis, or subdural haematoma
Endocrine disorders such as thyroid dysfunction or Cushing's syndrome
Urological disorders such as urinary retention
Gastrointestinal disorders such as hepatic failure, constipation (including faecal impaction), or
malnutrition
Severe uncontrolled pain
Alcohol intoxication or withdrawal
Medication e.g. opioids, benzodiazepines (use and withdrawal), dihydropyridines (such as
amlodipine), histamine-2 receptor antagonists, anti-Parkinsonian medications, tricyclic
antidepressants, lithium, antipsychotics, anticonvulsants, antiarrhythmics, antihypertensives,
histamine-2 receptor antagonists, corticosteroids, and NSAIDs
Psychosocial factors such as depression, sleep deprivation, visual or hearing impairment,
emotional stress, or change of environment
Complications
The complications of delirium include:
Increased mortality
Increased length of stay in hospital
Nosocomial infections
Increased risk of admission to long-term care or re-admission to hospital
Increased incidence of dementia
Falls
Pressure sores
Continence problems
Malnutrition
Functional impairment
Distress for the person, their family, and/or carers

Clinical features
Suspect delirium in people with a sudden change in behaviour that may be reported by the person, a
carer, or relative. Behavioural changes may include:
Altered cognitive function — the person may be disoriented, have memory and language
impairment, worsened concentration, slow responses, and confusion. The person may not be
able to recall details of their current illness, instructions, or names.
Inattention — the person may be easily distractible and have difficulty focusing and moving
attention from one thing to another, for example they are unable to maintain a conversation or
follow reasonable commands.
Disorganised thinking — the person may have disorganised, rambling, or irrelevant conversation,
unclear or illogical flow of ideas, and difficulty expressing their needs and concerns.
Altered perception — the person may experience paranoid delusions, misperceptions or, visual or
auditory hallucinations which may be distressing.
Altered physical function:
Hyperactive delirium — the person may have increased sensitivity to their immediate
surroundings with agitation, restlessness, sleep disturbance, and hypervigilance.
Restlessness and wandering are common.
Hypoactive delirium (more common) — the person may be lethargic, have reduced mobility
and movement, lack interest in daily activities, have a reduced appetite, and become quiet
and withdrawn.
Mixed — the person will have a combination of signs and symptoms of hyperactive and
hypoactive subtypes.
Altered social behaviour — the person may have intermittent and labile changes in mood and/or
emotions (such as fear, paranoia, anxiety, depression, irritability, apathy, anger, or euphoria). Their
behaviour may be inappropriate and they may not cooperate with reasonable requests or become
withdrawn.
Altered level of consciousness — the person may have a clouding of consciousness, reduced
awareness of their surroundings, and sleep-cycle disturbances (such as daytime drowsiness,
night time insomnia, disturbed sleep, or complete sleep cycle reversal). Impaired consciousness
can be subtle, and may initially only be apparent as lethargy or distractibility.

Differential diagnosis
Depression - Depression can present with similar symptoms to delirium (especially hypoactive
delirium), for example mood change, anorexia, sleep disturbance, and psychomotor change.
Dementia - Apart from Lewy-body dementia (which has fluctuation in cognition as one of its
features), dementia usually has an insidious onset and is not associated with fluctuations in
mental state.
Mental illness - Late-onset mania or schizophrenia can present with features similar to
hyperactive delirium.
Anxiety - Behavioural changes associated with delirium can include anxiety.
Thyroid disease - Hyperthyroidism and hypothyroidism can have similar features to hyperactive
delirium and hypoactive delirium respectively.
Non-convulsive epilepsy or temporal lobe epilepsy - Non-convulsive epilepsy can present with
subtle behaviour or mood changes and clouding of consciousness.
Charles Bonnet syndrome - Visual hallucinations may occur in people with severe visual
impairment and can range from simple patterns of straight lines to detailed pictures of people or
buildings.

Assessment
Delirium is a clinical diagnosis based on a detailed history, examination, and relevant investigations.
Take a history from the person and an informed observer (family member or carer).
Check vital signs including temperature, blood pressure, heart rate, capillary refill time, finger-
prick blood glucose, and pulse oximetry — to identify fever, hypoperfusion, hyperglycaemia,
hypoglycaemia, or hypoxia.
Carry out a general examination to identify precipitating factors.
Confirm a diagnosis of delirium by carrying out a cognitive assessment.
Arrange targeted investigations based on findings from the history and examination, for example:
Urinalysis — to identify conditions such as infection or hyperglycaemia. Arrange a mid-
stream urine (MSU) if urinalysis is abnormal.
Sputum culture — to identify chest infection.
Full blood count — to identify infection or anaemia.
Folate and B12 — to identify vitamin deficiency.
Urea and electrolytes — to identify acute kidney injury and electrolyte disturbance (such as
hyponatraemia or hypokalaemia).
HbA1c — to identify hyperglycaemia.
Calcium — to identify hypercalcaemia or hypocalcaemia.
Liver function tests — to identify hepatic failure and rule out hepatic encephalopathy.
Inflammatory markers (such as erythrocyte sedimentation rate and C-reactive protein) —
these tests are non-discriminatory but can help to identify infection or inflammation.
Drug levels — to identify drug toxicity, for example if the person has taken digoxin, lithium,
or alcohol.
Thyroid function tests — to identify hyperthyroidism or hypothyroidism.
Chest X-ray — to identify conditions such as pneumonia and heart failure.
Electrocardiogram — to identify cardiac conditions including arrhythmias.

Management
Most people with delirium should be admitted to hospital for urgent assessment, close
monitoring, and treatment. The decision as to whether to admit a person with delirium depends
on the person's specific clinical and social situation, and should also take into account the views
of family members or carers. If the person is deemed to not have capacity to consent, decisions
should be made in the best interests of the person using the Mental Capacity Act 2005. If the
person with delirium refuses admission ask carers or family (if appropriate) to help persuade the
person. If this fails, consider admission under the Mental Capacity Act (2005).
Management of a person with delirium includes:
Correcting any precipitating factors e.g. infection, drugs, constipation, urinary retention,
dehydration and electrolyte imbalance, pain
Optimising treatment of comorbidities
Managing behaviour change
Trying reorientation strategies e.g. regular cues, continuity of care from carers and
staff
Maintaining safe mobility e.g. avoiding physical restraint, encouraging walking
Normalising the sleep-wake cycle e.g. discouraging napping and encouraging
uninterrupted sleep at night
Managing challenging behaviour (such as aggression, agitation or shouting)
Addressing any underlying causes for the behaviour (such as discomfort, thirst, or
need for the toilet).
Moving the person to a safe, low-stimulation environment (such as a quiet room).
Using verbal and non-verbal de-escalation techniques (such as active listening,
effective verbal responding, pictures, and symbols).
Pharmacological measures
Specialists may suggest pharmacological measures as a last resort for severe
agitation or psychosis if:
Verbal and non-verbal de-escalation techniques are inappropriate or have
failed, and
The person is a danger to themselves or others, and
The cause of delirium is known and being treated, and
The benefit outweighs the risk to the person, and
There is enough care in place for the person to be continually monitored.
The following medication may be suggested:
Short-term (for 1 week or less) low-dose haloperidol (off-label indication).
Low-dose lorazepam (off-label indication) as an alternative if haloperidol is
contraindicated (for example in people with Parkinson's disease/parkinsonism,
Lewy-body dementia, or a prolonged QT interval).
Start at the lowest clinically appropriate dose and titrate cautiously according to
symptoms. The aim of drug treatment is to calm (not sedate) the person.
Elderly Care & Fragility  QUESTION 4 OF 12

A 69 year old woman is brought to the Emergency Department after being found
driving the wrong way down a motorway. She was stopped by the police before any
collision occurred. She is known to the police officers as she is often found wandering
the streets at night. You suspect a diagnosis of dementia. Which of the following
investigations is NOT routinely recommended for the investigation of suspected
dementia?

B12 and folate 3%

CT or MRI head 12%

Serum calcium levels 11%

Thyroid function tests 5%

✓ Syphilis serology 70%

ANSWER

To help identify reversible causes of dementia and exclude other causes of symptoms initial
investigations include:
In most cases:
Full blood count
Erythrocyte sedimentation rate (ESR)
Urea and electrolytes
Calcium
HbA1c
Liver function tests
Thyroid function tests
Serum B12 and folate levels
If clinically indicated:
A midstream urine, for example when delirium is a possibility
Chest X-ray, electrocardiogram (ECG), syphilis serology, and HIV testing
Imaging
Should include structural imaging (MRI or CT scan) to exclude non-dementia cerebral
pathology such as normal pressure hydrocephalus and to identify dementia subtype
Dementia LAST UPDATED: 12TH
FEBRUARY 2021
ELDERLY CARE & FRAGILITY
 Bookmark

Dementia is a typically progressive clinical syndrome of deteriorating mental function significant


enough to interfere with activities of daily living (ADLs). It affects cognitive domains (such as memory,
thinking, language, orientation and judgement) and social behaviour (such as emotional control and
motivation).

Causes
Alzheimer’s disease (50 – 75% of cases)
Atrophy of the cerebral cortex
Formation of amyloid plaques and neurofibrillary tangles
Reduced neuronal acetylcholine production
Vascular dementia (up to 20% of cases)
Reduced blood supply to the brain
Dementia with Lewy bodies (DLB) (10 – 15% of cases)
Cortical and subcortical Lewy bodies (abnormal deposits of protein inside nerve cells)
Frontotemporal dementia (FTB) (2% of cases)
Progressive degeneration of the frontal and/or temporal lobe
Rarer causes of dementia:
Parkinson’s disease dementia
Progressive supranuclear palsy
Huntington’s disease
Prion disease (such as Creutzfeldt-Jakob disease [CJD])
Normal pressure hydrocephalus (NPH)
Chronic subdural haematoma
Benign tumours
Metabolic and endocrine disorders (such as chronic hypocalcaemia and recurrent
hypoglycaemia)
Vitamin deficiencies (such as B12 and thiamine deficiency)
Infections (such as HIV infection and syphilis)

Risk factors
Age - strongest risk factor for dementia
Mild cognitive impairment
Learning difficulties
Genetics
Cardiovascular disease risk factors (such as diabetes, smoking, hypercholesterolemia, and
hypertension)
Parkinson's disease
Stroke
Depression
Heavy alcohol consumption
Low educational attainment
Low social engagement and support

Clinical features
Dementia can be difficult to identify as it usually has an insidious onset and non-specific signs and
symptoms, which vary from person to person. People with early dementia may deny symptoms or
accommodate to cognitive change and functional ability.
Suspect dementia if any of the following are reported by the person and/or their family/carer:
Cognitive impairment, including:
Memory problems
Receptive or expressive dysphasia
Difficulty in carrying out coordinated movements such as dressing
Disorientation and unawareness of the time and place
Impairment of executive function, such as difficulties with planning and problem solving
Behavioural and psychological symptoms of dementia (BPSD) tend to fluctuate, may last for 6
months or more and include:
Psychosis
Agitation and emotional lability
Depression and anxiety
Withdrawal or apathy
Disinhibition
Motor disturbance
Sleep cycle disturbance or insomnia
Tendency to repeat phrases or questions
Difficulties with activities of daily living (ADLs):
In the early stages of dementia this may lead to neglect of household tasks, nutrition
(causing weight loss), personal hygiene, and grooming. People with dementia who are in
employment may find that they are increasingly making mistakes at work.
In the later stages, basic ADLs such as dressing, eating, and walking become affected.
Specific features of subtypes of dementia:
For Alzheimer’s disease:
Early impairment of episodic memory — this may include memory loss for recent events,
repeated questioning, and difficulty learning new information.
For vascular dementia:
Stepwise increases in the severity of symptoms — subcortical ischaemic vascular dementia
may present insidiously with gait and attention problems and changes in personality.
Focal neurological signs (such as hemiparesis or visual field defects) may be present.
For dementia with Lewy bodies:
Repeated falls, syncope or transient loss of consciousness, severe sensitivity to
antipsychotics, delusions, and hallucinations may be present. Memory impairment may not
be apparent in early stages. Parkinsonian motor features (such as shuffling gait, rigidity,
bradykinesia, and loss of spontaneous movement) and autonomic dysfunction (such as
postural hypotension, difficulty in swallowing, and incontinence or constipation) may be
present.
For frontotemporal dementia (FTD):
Personality change and behavioural disturbance (such as apathy or social/sexual
disinhibition) may develop insidiously. Other cognitive functions (such as memory and
perception) may be relatively preserved.

Differential diagnosis
Conditions that can present with similar symptoms to dementia include:
Normal-age related memory changes - Normal ageing is associated with a mild decline in
cognitive function, and memory lapses are common, especially during times of physical illness or
stress.
Mild cognitive impairment (MCI) - MCI differs from dementia in that symptoms do not fulfil the
diagnostic criteria for dementia, for example only one cognitive domain may be affected or
activities of daily life may not be significantly affected. Of people with MCI, 50% will later develop
dementia.
Depression - Symptoms of depression include low mood, loss of interest, anhedonia, and self-
neglect which can be similar to those of dementia.In older people, features of depression may be
less obvious, with somatic symptoms (such as reduced appetite, fatigue, and insomnia) are more
common.
Delirium - Delirium is an acute, fluctuating syndrome of disturbed consciousness, attention,
cognition, and perception. It is a common condition in the differential diagnosis for dementia.
People with cognitive impairment are at increased risk of delirium, and the two conditions often
coexist.
Vitamin deficiency - Thiamine deficiency can lead to Wernicke encephalopathy and Korsakoff
psychosis. Symptoms include confusion, memory loss, problems with learning new information
and gait disturbances. Vitamin B12 deficiency can lead to ataxia, psychiatric abnormalities,
memory loss, and gait disturbance.
Hypothyroidism - Symptoms of hypothyroidism can include low mood, and impaired
concentration and memory.
Adverse drug effects - Many drugs, including benzodiazepines, analgesics (such as opioids,
naproxen, and ibuprofen), anticholinergics, antidepressants (such as tricyclics), antipsychotics
(such as haloperidol), anticonvulsants (especially older preparations, such as phenytoin and
phenobarbital), and corticosteroids can affect cognition.
Normal pressure hydrocephalus - Normal pressure hydrocephalus can present with symptoms of
early cognitive impairment, urinary incontinence, and gait disorder. In NPH, cerebrospinal fluid
(CSF) flow has been impeded but compensatory mechanisms have prevented raised pressure.
The lateral ventricles are prominently dilated, and these ventricles exert local pressure on certain
brain areas to give the classical clinical picture. Pressure on the frontal lobes gives the dementia
and pressure on the medial side of the motor cortex, and the pyramidal tract fibres, cause
incontinence and pyramidal leg weakness. Diagnosis is by lumbar puncture (to demonstrate a
normal CSF opening pressure) followed by head computed tomography (CT)/magnetic resonance
imaging (MRI) (showing enlarged ventricles). Treatment is with ventriculoperitoneal shunting.
Sensory deficits - Problems with vision and hearing can contribute significantly to an apparent
decline in cognitive ability.

Investigations
Patient's should be assessed using a standardised cognitive assessment tool.
To help identify reversible causes of dementia and exclude other causes of symptoms initial
investigations include:
In most cases:
Full blood count
Erythrocyte sedimentation rate (ESR)
Urea and electrolytes
Calcium
HbA1c
Liver function tests
Thyroid function tests
Serum B12 and folate levels
If clinically indicated:
A midstream urine, for example when delirium is a possibility
Chest X-ray, electrocardiogram (ECG), syphilis serology, and HIV testing
Patients with suspected dementia should be referred to a specialist memory assessment service
for further specialist assessment and management.
Specialist investigations for suspected dementia:
Should include structural imaging (MRI or CT scan) to: exclude non-dementia cerebral
pathology such as normal pressure hydrocephalus and to identify dementia subtype.
May also include:
HMPAO SPECT and FDG PET as second line investigations if diagnosis or subtype is
unclear
(FP-CIT) SPECT where a potential diagnosis of dementia with Lewy bodies (DLB) is in
doubt
Cerebrospinal fluid examination to exclude inflammatory, infective, or malignant
causes of dementia, in cases where dementia is rapidly progressive, the presentation
is unusual, or the person is younger than 55 years of age
Electroencephalography if a diagnosis of delirium, frontotemporal dementia, or
Creutzfeldt–Jakob disease is suspected, or in the assessment of associated seizure
disorder in people with dementia
Brain biopsy in highly selected people whose dementia is thought to be due to a
potentially reversible condition (such as cerebral vasculitis) that cannot be
diagnosed in another way

Management
Specialist non-pharmacological interventions for cognitive symptoms of dementia:
Structured group cognitive stimulation programs
Reminiscence therapy with discussion of past experiences
Tools such as life histories, shared memories, and familiar objects from the past
Specialist drug treatments for cognitive symptoms of dementia:
Acetylcholinesterase (AChE) inhibitors (donepezil, galantamine, and rivastigmine) - These
drugs can be used for mild to moderate Alzheimer's disease.
Memantine (a N-methyl-D-aspartic acid receptor antagonist) - Memantine is an option for
managing Alzheimer's disease for people with moderate Alzheimer's disease who are
intolerant of, or have a contraindication to, AChE inhibitors or in severe Alzheimer's disease.
Specialist drug treatment for behavioural and psychological symptoms of dementia (BPSD) or
non-cognitive symptoms of dementia and challenging behaviour:
No antipsychotics (except risperidone in very specific circumstances) are licensed for
BPSD in the UK. Specialists may prescribe an antipsychotic drug off-label in certain
circumstances (e.g. if they are severely distressed or there is an immediate risk of harm to
the person or others) following a thorough clinical assessment and discussion (including
the risks and benefits) with the carer and (if possible) the person with dementia.
Antipsychotics have potentially serious adverse effects, including increased risk of stroke
and mortality, Parkinsonism, and cognitive impairment. They should be used with caution,
at low dose, and for the shortest time possible.
In specific situations, specialists may offer AChE inhibitors for BPSD if non-
pharmacological methods and anti-psychotic drugs are inappropriate or ineffective.
Elderly Care & Fragility  QUESTION 5 OF 12

A 67 year old man is brought to the Emergency Department by paramedics after


being found confused in a nearby supermarket. The staff in the store reported seeing
him most days and found he was occasionally slightly "muddled" but never struggled
like today. He was unaware of his surroundings and was shouting at members of the
public. His observations are recorded as:
Heart rate: 65 beats per minute
Blood pressure: 109/65 mmHg
Respiratory rate: 10 breaths per minute
Oxygen saturations: 94% on air
Temperature: 37.0°C
He opens his eyes and localises to pain. He is unable to follow commands and his
speech is confused. His examination is otherwise normal. Intravenous access is
obtained. What is the most appropriate next management step?

Check arterial blood gases 4%

✓ Check bedside serum glucose 85%

Give 10% glucose IV 2%

Give naloxone 400 micrograms IV 7%

x Perform electrocardiogram 1%

ANSWER
All patients with altered mental status should have a rapid determination of their glucose. Although this
patient has several abnormal vital signs, none necessitate immediate action. His Glasgow Coma Score
(GCS) is 11, and it appears that his respiratory status is adequate. Dextrose should not be given unless
hypoglycaemia is documented. Naloxone is an opioid antagonist, and it could reverse the patient’s
confusion if it is caused by opioids. This drug could be considered once other more easily reversible causes
of confusion (e.g. hypoglycemia) have been ruled out. ABG and ECG are both occasionally needed in
patients with altered mental status, but neither is as urgent as glucose testing.
Delirium LAST UPDATED: 12TH
FEBRUARY 2021
ELDERLY CARE & FRAGILITY
 Bookmark

Delirium (sometimes called 'acute confusional state') is an acute, fluctuating syndrome of disturbed
consciousness, attention, cognition and perception.
Delirium usually develops acutely over hours to days and behavioural disturbance, personality changes,
and psychotic features may occur. Symptoms typically fluctuate (come and go or increase and
decrease in severity). Lucid intervals usually occur during the day with the worst disturbance at night.
Delirium typically occurs in people with predisposing factors (such as advanced age or multiple
comorbidities) when new precipitating factors (such as some medications or infection) are added.

Predisposing factors
Predisposing factors include:
Older age (over 65 years)
Cognitive impairment (such as dementia)
Frailty/multiple comorbidities (such as stroke or heart failure)
Significant injuries such as hip fracture
Functional impairment (for example immobility or the use of physical restraints such as cot sides)
Iatrogenic events (such as bladder catheterisation, polypharmacy, or surgery)
History of, or current, alcohol excess
Sensory impairment (such as visual impairment or hearing loss)
Poor nutrition
Lack of stimulation
Terminal phase of illness

Precipitating factors
Precipitating factors include:
Infection such as urinary tract infection, infected pressure sore, or pneumonia
Metabolic disturbance such as hypoglycaemia, hyperglycaemia, or electrolyte abnormalities
Cardiovascular disorders such as myocardial infarction or heart failure
Respiratory disorders such as pulmonary embolism or exacerbation of chronic obstructive
pulmonary disease
Neurological disorders such as stroke, encephalitis, or subdural haematoma
Endocrine disorders such as thyroid dysfunction or Cushing's syndrome
Urological disorders such as urinary retention
Gastrointestinal disorders such as hepatic failure, constipation (including faecal impaction), or
malnutrition
Severe uncontrolled pain
Alcohol intoxication or withdrawal
Medication e.g. opioids, benzodiazepines (use and withdrawal), dihydropyridines (such as
amlodipine), histamine-2 receptor antagonists, anti-Parkinsonian medications, tricyclic
antidepressants, lithium, antipsychotics, anticonvulsants, antiarrhythmics, antihypertensives,
histamine-2 receptor antagonists, corticosteroids, and NSAIDs
Psychosocial factors such as depression, sleep deprivation, visual or hearing impairment,
emotional stress, or change of environment

Complications
The complications of delirium include:
Increased mortality
Increased length of stay in hospital
Nosocomial infections
Increased risk of admission to long-term care or re-admission to hospital
Increased incidence of dementia
Falls
Pressure sores
Continence problems
Malnutrition
Functional impairment
Distress for the person, their family, and/or carers

Clinical features
Suspect delirium in people with a sudden change in behaviour that may be reported by the person, a
carer, or relative. Behavioural changes may include:
Altered cognitive function — the person may be disoriented, have memory and language
impairment, worsened concentration, slow responses, and confusion. The person may not be
able to recall details of their current illness, instructions, or names.
Inattention — the person may be easily distractible and have difficulty focusing and moving
attention from one thing to another, for example they are unable to maintain a conversation or
follow reasonable commands.
Disorganised thinking — the person may have disorganised, rambling, or irrelevant conversation,
unclear or illogical flow of ideas, and difficulty expressing their needs and concerns.
Altered perception — the person may experience paranoid delusions, misperceptions or, visual or
auditory hallucinations which may be distressing.
Altered physical function:
Hyperactive delirium — the person may have increased sensitivity to their immediate
surroundings with agitation, restlessness, sleep disturbance, and hypervigilance.
Restlessness and wandering are common.
Hypoactive delirium (more common) — the person may be lethargic, have reduced mobility
and movement, lack interest in daily activities, have a reduced appetite, and become quiet
and withdrawn.
Mixed — the person will have a combination of signs and symptoms of hyperactive and
hypoactive subtypes.
Altered social behaviour — the person may have intermittent and labile changes in mood and/or
emotions (such as fear, paranoia, anxiety, depression, irritability, apathy, anger, or euphoria). Their
behaviour may be inappropriate and they may not cooperate with reasonable requests or become
withdrawn.
Altered level of consciousness — the person may have a clouding of consciousness, reduced
awareness of their surroundings, and sleep-cycle disturbances (such as daytime drowsiness,
night-time insomnia, disturbed sleep, or complete sleep cycle reversal). Impaired consciousness
can be subtle, and may initially only be apparent as lethargy or distractibility.

Differential diagnosis
Depression - Depression can present with similar symptoms to delirium (especially hypoactive
delirium), for example mood change, anorexia, sleep disturbance, and psychomotor change.
Dementia - Apart from Lewy-body dementia (which has fluctuation in cognition as one of its
features), dementia usually has an insidious onset and is not associated with fluctuations in
mental state.
Mental illness - Late-onset mania or schizophrenia can present with features similar to
hyperactive delirium.
Anxiety - Behavioural changes associated with delirium can include anxiety.
Thyroid disease - Hyperthyroidism and hypothyroidism can have similar features to hyperactive
delirium and hypoactive delirium respectively.
Non-convulsive epilepsy or temporal lobe epilepsy - Non-convulsive epilepsy can present with
subtle behaviour or mood changes and clouding of consciousness.
Charles Bonnet syndrome - Visual hallucinations may occur in people with severe visual
impairment and can range from simple patterns of straight lines to detailed pictures of people or
buildings.

Assessment
Delirium is a clinical diagnosis based on a detailed history, examination, and relevant investigations.
Take a history from the person and an informed observer (family member or carer).
Check vital signs including temperature, blood pressure, heart rate, capillary refill time, finger-
prick blood glucose, and pulse oximetry — to identify fever, hypoperfusion, hyperglycaemia,
hypoglycaemia, or hypoxia.
Carry out a general examination to identify precipitating factors.
Confirm a diagnosis of delirium by carrying out a cognitive assessment.
Arrange targeted investigations based on findings from the history and examination, for example:
Urinalysis — to identify conditions such as infection or hyperglycaemia. Arrange a mid-
stream urine (MSU) if urinalysis is abnormal.
Sputum culture — to identify chest infection.
Full blood count — to identify infection or anaemia.
Folate and B12 — to identify vitamin deficiency.
Urea and electrolytes — to identify acute kidney injury and electrolyte disturbance (such as
hyponatraemia or hypokalaemia).
HbA1c — to identify hyperglycaemia.
Calcium — to identify hypercalcaemia or hypocalcaemia.
Liver function tests — to identify hepatic failure and rule out hepatic encephalopathy.
Inflammatory markers (such as erythrocyte sedimentation rate and C-reactive protein) —
these tests are non-discriminatory but can help to identify infection or inflammation.
Drug levels — to identify drug toxicity, for example if the person has taken digoxin, lithium,
or alcohol.
Thyroid function tests — to identify hyperthyroidism or hypothyroidism.
Chest X-ray — to identify conditions such as pneumonia and heart failure.
Electrocardiogram — to identify cardiac conditions including arrhythmias.

Management
Most people with delirium should be admitted to hospital for urgent assessment, close
monitoring, and treatment. The decision as to whether to admit a person with delirium depends
on the person's specific clinical and social situation, and should also take into account the views
of family members or carers. If the person is deemed to not have capacity to consent, decisions
should be made in the best interests of the person using the Mental Capacity Act 2005. If the
person with delirium refuses admission ask carers or family (if appropriate) to help persuade the
person. If this fails, consider admission under the Mental Capacity Act (2005).
Management of a person with delirium includes:
Correcting any precipitating factors e.g. infection, drugs, constipation, urinary retention,
dehydration and electrolyte imbalance, pain
Optimising treatment of comorbidities
Managing behaviour change
Trying reorientation strategies e.g. regular cues, continuity of care from carers and
staff
Maintaining safe mobility e.g. avoiding physical restraint, encouraging walking
Normalising the sleep-wake cycle e.g. discouraging napping and encouraging
uninterrupted sleep at night
Managing challenging behaviour (such as aggression, agitation or shouting)
Addressing any underlying causes for the behaviour (such as discomfort, thirst, or
need for the toilet).
Moving the person to a safe, low-stimulation environment (such as a quiet room).
Using verbal and non-verbal de-escalation techniques (such as active listening,
effective verbal responding, pictures, and symbols).
Pharmacological measures
Specialists may suggest pharmacological measures as a last resort for severe
agitation or psychosis if:
Verbal and non-verbal de-escalation techniques are inappropriate or have
failed, and
The person is a danger to themselves or others, and
The cause of delirium is known and being treated, and
The benefit outweighs the risk to the person, and
There is enough care in place for the person to be continually monitored.
The following medication may be suggested:
Short-term (for 1 week or less) low-dose haloperidol (off-label indication).
Low-dose lorazepam (off-label indication) as an alternative if haloperidol is
contraindicated (for example in people with Parkinson's disease/parkinsonism,
Lewy-body dementia, or a prolonged QT interval).
Start at the lowest clinically appropriate dose and titrate cautiously according to
symptoms. The aim of drug treatment is to calm (not sedate) the person.
Elderly Care & Fragility  QUESTION 6 OF 12

A 78 year old woman presents to the Emergency Department following a fall. She has
a deformed, painful right wrist. Her wrist x-ray shows a Colles' fracture and you
consider if she has underlying osteoporosis. Which of the following is a risk factor for
osteoporosis?

Male gender 1%

Obesity 4%

Excessive exercise 1%

Long term use of NSAIDs 6%

✓ Oestrogen deficiency 88%

ANSWER
Risk factors for having a fragility fracture:
Risks factors affecting bone strength that reduce BMD:
Endocrine disease e.g. diabetes mellitus, hyperthyroidism, hyperparathyroidism
Malabsorptive disease e.g. IBD, coeliac disease, chronic pancreatitis
Chronic kidney disease
Chronic liver disease
COPD
Menopause
Immobility
BMI < 18.5 kg/m2
Risks factors affecting bone strength that do not reduce BMD:
Age
Oral corticosteroids
Smoking
Alcohol
Previous fragility fracture
Rheumatological disease e.g. rheumatoid arthritis
Parental history of hip fracture
Risk factors affecting bone strength with unestablished mechanisms:
Selective serotonin reuptake inhibitors (SSRIs)
Proton pump inhibitors (PPI)
Anticonvulsant drugs e.g. carbamazepine
Risk factors for falls:
Impaired vision
Neuromuscular weakness and incoordination
Cognitive impairment
Use of alcohol and sedative drugs

Osteoporosis and Fragility Fractures LAST UPDATED: 5TH MARCH


2021
ELDERLY CARE & FRAGILITY
 Bookmark

Osteoporosis is a disease characterised by low bone mass and structural deterioration of bone tissue,
with a consequent increase in bone fragility and susceptibility to fracture. Osteoporosis is the end
result of an imbalance in the normal process of bone remodelling by osteoclasts and osteoblasts. It is
asymptomatic and often remains undiagnosed until a fragility fracture occurs.

Definitions
Osteoporosis is defined by the World Health Organization as a bone mineral density (BMD) of 2.5
standard deviations below the mean peak mass (average of young healthy adults) as measured
by dual-energy X-ray absorptiometry (DXA) applied to the femoral neck and reported as a T-
score.
An osteoporotic fracture is a fragility fracture occurring as a consequence of osteoporosis.
Osteoporotic fractures characteristically occur in the wrist, spine and hip, but can also occur in
the arm, pelvis, ribs and other bones.
A fragility fracture is defined as a fracture following a fall from standing height or less, although
vertebral fractures may occur spontaneously, or as a result of routine activities such as bending
or lifting.

Risk factors
The risk of getting an osteoporotic fracture depends on the person's risk of falls, their bone strength
(determined by bone mineral density), and other risk factors.
Risk factors for having a fragility fracture:
Risks factors affecting bone strength that reduce BMD:
Endocrine disease e.g. diabetes mellitus, hyperthyroidism, hyperparathyroidism
Malabsorptive disease e.g. IBD, coeliac disease, chronic pancreatitis
Chronic kidney disease
Chronic liver disease
COPD
Menopause
Immobility
BMI < 18.5 kg/m2
Risks factors affecting bone strength that do not reduce BMD:
Age
Oral corticosteroids
Smoking
Alcohol
Previous fragility fracture
Rheumatological disease e.g. rheumatoid arthritis
Parental history of hip fracture
Risk factors affecting bone strength with unestablished mechanisms:
Selective serotonin reuptake inhibitors (SSRIs)
Proton pump inhibitors (PPI)
Anticonvulsant drugs e.g. carbamazepine
Risk factors for falls:
Impaired vision
Neuromuscular weakness and incoordination
Cognitive impairment
Use of alcohol and sedative drugs

Assessment for fragility fracture risk


Exclude non-osteoporotic causes for fragility fractures:
Metastatic bone disease (bone pain, history of cancer (especially lung, thyroid, prostate,
kidney, or breast cancer), or symptoms of undiagnosed cancer (for example unexplained
general malaise or weight loss)
Multiple myeloma (bone pain, anaemia, recurrent infection, bleeding, hypercalcaemia,
kidney disease)
Osteomalacia (bone pain, muscle pain or proximal muscle weakness)
Paget’s disease (bone pain or deformity)
Aneurysmal bone cyst or hemangioma
Infection e.g. osteomyelitis, Pott's disease
Exclude secondary causes of osteoporosis especially in people with a fragility fracture who are at
low risk, including men of any age, pre-menopausal women, and women with premature
menopause (under 40 years of age)
Consider starting drug treatment in people with vertebral or hip fractures without undertaking
DXA if this is considered inappropriate or impractical
Offer a dual-energy X-ray absorptiometry (DXA) scan to measure bone mineral density (BMD)
without calculating the fragility fracture risk in people:
Over 50 years of age with a history of fragility fracture
Younger than 40 years of age who have a major risk factor for fragility fracture
For all other people with risk factors for osteoporosis, calculate the 10-year fragility fracture risk
prior to arranging a DXA scan to measure BMD; consider using the online risk calculators
QFracture® (preferred) or FRAX®, which predict the absolute risk of hip fracture and major
osteoporotic fractures (spine, wrist, hip, or shoulder) over 10 years
High risk: For people whose fracture risk is above the recommended threshold, offer a dual-
energy x-ray absorptiometry (DXA) scan, then bone-sparing drug treatment if the T-score
is -2.5 or less; if the T-score is greater than -2.5, modify risk factors where possible, treat
any underlying conditions, and repeat the DXA at an interval appropriate for the person
based on their risk profile, using clinical judgement (but usually within 2 years)
Intermediate risk: For people whose fracture risk is close to the recommended threshold
and who have risk factors that may be underestimated by FRAX®, arrange a DXA scan to
measure their bone mineral density (BMD) and offer drug treatment if the T-score is -2.5 or
less
Low risk: For people whose fracture risk is below the recommended threshold, do not offer
drug treatment, offer lifestyle advice and follow up within 5 years
Assess for vitamin D deficiency and inadequate calcium intake.
People are at risk of vitamin D deficiency if they are aged over 65 years, or are not exposed
to much sunlight (because they are confined indoors for long periods, or because they wear
clothes that cover the whole body)
A calcium intake of at least 1000 mg/day is recommended for people at increased risk of a
fragility fracture
Identify any risk factors for falls

Management
Lifestyle advice
Advise the person to:
Take regular exercise to improve muscle strength
Eat a balanced diet
Stop smoking if needed
Drink alcohol within recommended limits
Bone-sparing treatment
Prescribe a bisphosphonate (alendronate 10 mg once daily or 70 mg once weekly, or
risedronate 5 mg once daily or 35 mg once weekly), if there are no contraindications and
after appropriate counselling, to postmenopausal women and men over 50 years of age
who have been confirmed by dual-energy X-ray absorptiometry (DXA) scan to have
osteoporosis (bone mineral density [BMD] T-score of -2.5 or less).
Consider prescribing to people who are taking high doses of oral corticosteroids (more than
or equivalent to prednisolone 7.5 mg daily for 3 months or longer).
If an oral bisphosphonate is not tolerated or is contraindicated, consider specialist referral.
Specialist treatment options include zoledronic acid, strontium ranelate, raloxifene,
denosumab, and teriparatide.
Calcium and vitamin D supplements
If the person's calcium intake is adequate (700 mg/day), prescribe 10 micrograms (400
international units) of vitamin D (without calcium) for people not exposed to much sunlight.
If calcium intake is inadequate prescribe 10 micrograms (400 international units) of vitamin
D with at least 1000 mg of calcium daily; prescribe 20 micrograms (800 international units)
of vitamin D with at least 1000 mg of calcium daily for elderly people who are housebound
or living in a nursing home.
Hormone replacement therapy
Consider prescribing hormone replacement therapy (HRT) to women who have a premature
menopause (menopause before 40 years of age) to reduce the risk of fragility fractures and
for the relief of menopausal symptoms.

Vertebral compression fractures


Vertebral compression fractures are the most common type of osteoporotic fracture. They often occur
at the midthoracic (T7 - T8) spine and the thoracolumbar junction (T12 - L1).
Fractures may result in significant back pain, limited physical functioning and activities of daily living,
and can lead to loss of independence, depression, and chronic pain. Osteoporotic vertebral
compression that occurs slowly over time is often asymptomatic. Old or healed fractures may be an
incidental finding on radiographs of the chest or abdomen. In other patients, the presence of vertebral
fractures may become apparent because of height loss or kyphosis. In contrast, acute episodes of
significant vertebral body compression are associated with pain.
In patients who have acute symptomatic vertebral body fracture, there is often no history of preceding
trauma. The typical patient presents with acute back pain after sudden bending, coughing, or
lifting. The pain is usually well localised to the midline spine but often refers in a unilateral or bilateral
pattern into the flank, anterior abdomen, or the posterior superior iliac spine. On physical examination,
the patient may experience pain upon palpation and percussion of the corresponding spinous process
and paravertebral structures.
Acute episodes of pain following a vertebral body fracture usually resolve after four to six weeks, but
pain may persist for longer periods (many months), indicating an unhealed or slowly healing
fracture. The initial management of osteoporotic vertebral compression fractures should include pain
control and activity modification (bed rest is not recommended). The indications for and timing of
vertebral augmentation procedures are controversial. Most patients with vertebral compression
fractures can be treated successfully with conservative management.
Elderly Care & Fragility  QUESTION 7 OF 12

A 67 year old man is brought to the Emergency Department by his wife in the middle
of the night. She tells you that over the last year he has become increasingly
confused and often wanders at night. She is worried for his safety and on this
occasion he did not recognise his wife. His wife has come to the conclusion he most
likely has dementia. Which of the following is the most common cause of dementia?

Vascular dementia 9%

Fronotemporal dementia 3%

✓ Alzheimer's disease 81%

Lewy body dementia 5%

x Normal pressure hydrocephalus (NPH) Chronic subdural haematoma 1%

ANSWER
Causes of dementia:
Alzheimer’s disease (50 – 75% of cases)
Vascular dementia (up to 20% of cases)
Dementia with Lewy bodies (DLB) (10 – 15% of cases)
Frontotemporal dementia (FTB) (2% of cases)
Rarer causes of dementia:
Parkinson’s disease dementia
Progressive supranuclear palsy
Huntington’s disease
Prion disease (such as Creutzfeldt-Jakob disease [CJD])
Normal pressure hydrocephalus (NPH)
Chronic subdural haematoma
Benign tumours
Metabolic and endocrine disorders (such as chronic hypocalcaemia and recurrent
hypoglycaemia)
Vitamin deficiencies (such as B12 and thiamine deficiency)
Infections (such as HIV infection and syphilis)
Dementia LAST UPDATED: 12TH
FEBRUARY 2021
ELDERLY CARE & FRAGILITY
 Bookmark

Dementia is a typically progressive clinical syndrome of deteriorating mental function significant


enough to interfere with activities of daily living (ADLs). It affects cognitive domains (such as memory,
thinking, language, orientation and judgement) and social behaviour (such as emotional control and
motivation).

Causes
Alzheimer’s disease (50 – 75% of cases)
Atrophy of the cerebral cortex
Formation of amyloid plaques and neurofibrillary tangles
Reduced neuronal acetylcholine production
Vascular dementia (up to 20% of cases)
Reduced blood supply to the brain
Dementia with Lewy bodies (DLB) (10 – 15% of cases)
Cortical and subcortical Lewy bodies (abnormal deposits of protein inside nerve cells)
Frontotemporal dementia (FTB) (2% of cases)
Progressive degeneration of the frontal and/or temporal lobe
Rarer causes of dementia:
Parkinson’s disease dementia
Progressive supranuclear palsy
Huntington’s disease
Prion disease (such as Creutzfeldt-Jakob disease [CJD])
Normal pressure hydrocephalus (NPH)
Chronic subdural haematoma
Benign tumours
Metabolic and endocrine disorders (such as chronic hypocalcaemia and recurrent
hypoglycaemia)
Vitamin deficiencies (such as B12 and thiamine deficiency)
Infections (such as HIV infection and syphilis)

Risk factors
Age - strongest risk factor for dementia
Mild cognitive impairment
Learning difficulties
Genetics
Cardiovascular disease risk factors (such as diabetes, smoking, hypercholesterolemia, and
hypertension)
Parkinson's disease
Stroke
Depression
Heavy alcohol consumption
Low educational attainment
Low social engagement and support

Clinical features
Dementia can be difficult to identify as it usually has an insidious onset and non-specific signs and
symptoms, which vary from person to person. People with early dementia may deny symptoms or
accommodate to cognitive change and functional ability.
Suspect dementia if any of the following are reported by the person and/or their family/carer:
Cognitive impairment, including:
Memory problems
Receptive or expressive dysphasia
Difficulty in carrying out coordinated movements such as dressing
Disorientation and unawareness of the time and place
Impairment of executive function, such as difficulties with planning and problem solving
Behavioural and psychological symptoms of dementia (BPSD) tend to fluctuate, may last for 6
months or more and include:
Psychosis
Agitation and emotional lability
Depression and anxiety
Withdrawal or apathy
Disinhibition
Motor disturbance
Sleep cycle disturbance or insomnia
Tendency to repeat phrases or questions
Difficulties with activities of daily living (ADLs):
In the early stages of dementia this may lead to neglect of household tasks, nutrition
(causing weight loss), personal hygiene, and grooming. People with dementia who are in
employment may find that they are increasingly making mistakes at work.
In the later stages, basic ADLs such as dressing, eating, and walking become affected.
Specific features of subtypes of dementia:
For Alzheimer’s disease:
Early impairment of episodic memory — this may include memory loss for recent events,
repeated questioning, and difficulty learning new information.
For vascular dementia:
Stepwise increases in the severity of symptoms — subcortical ischaemic vascular dementia
may present insidiously with gait and attention problems and changes in personality.
Focal neurological signs (such as hemiparesis or visual field defects) may be present.
For dementia with Lewy bodies:
Repeated falls, syncope or transient loss of consciousness, severe sensitivity to
antipsychotics, delusions, and hallucinations may be present. Memory impairment may not
be apparent in early stages. Parkinsonian motor features (such as shuffling gait, rigidity,
bradykinesia, and loss of spontaneous movement) and autonomic dysfunction (such as
postural hypotension, difficulty in swallowing, and incontinence or constipation) may be
present.
For frontotemporal dementia (FTD):
Personality change and behavioural disturbance (such as apathy or social/sexual
disinhibition) may develop insidiously. Other cognitive functions (such as memory and
perception) may be relatively preserved.

Differential diagnosis
Conditions that can present with similar symptoms to dementia include:
Normal-age related memory changes - Normal ageing is associated with a mild decline in
cognitive function, and memory lapses are common, especially during times of physical illness or
stress.
Mild cognitive impairment (MCI) - MCI differs from dementia in that symptoms do not fulfil the
diagnostic criteria for dementia, for example only one cognitive domain may be affected or
activities of daily life may not be significantly affected. Of people with MCI, 50% will later develop
dementia.
Depression - Symptoms of depression include low mood, loss of interest, anhedonia, and self-
neglect which can be similar to those of dementia.In older people, features of depression may be
less obvious, with somatic symptoms (such as reduced appetite, fatigue, and insomnia) are more
common.
Delirium - Delirium is an acute, fluctuating syndrome of disturbed consciousness, attention,
cognition, and perception. It is a common condition in the differential diagnosis for dementia.
People with cognitive impairment are at increased risk of delirium, and the two conditions often
coexist.
Vitamin deficiency - Thiamine deficiency can lead to Wernicke encephalopathy and Korsakoff
psychosis. Symptoms include confusion, memory loss, problems with learning new information
and gait disturbances. Vitamin B12 deficiency can lead to ataxia, psychiatric abnormalities,
memory loss, and gait disturbance.
Hypothyroidism - Symptoms of hypothyroidism can include low mood, and impaired
concentration and memory.
Adverse drug effects - Many drugs, including benzodiazepines, analgesics (such as opioids,
naproxen, and ibuprofen), anticholinergics, antidepressants (such as tricyclics), antipsychotics
(such as haloperidol), anticonvulsants (especially older preparations, such as phenytoin and
phenobarbital), and corticosteroids can affect cognition.
Normal pressure hydrocephalus - Normal pressure hydrocephalus can present with symptoms of
early cognitive impairment, urinary incontinence, and gait disorder. In NPH, cerebrospinal fluid
(CSF) flow has been impeded but compensatory mechanisms have prevented raised pressure.
The lateral ventricles are prominently dilated, and these ventricles exert local pressure on certain
brain areas to give the classical clinical picture. Pressure on the frontal lobes gives the dementia
and pressure on the medial side of the motor cortex, and the pyramidal tract fibres, cause
incontinence and pyramidal leg weakness. Diagnosis is by lumbar puncture (to demonstrate a
normal CSF opening pressure) followed by head computed tomography (CT)/magnetic resonance
imaging (MRI) (showing enlarged ventricles). Treatment is with ventriculoperitoneal shunting.
Sensory deficits - Problems with vision and hearing can contribute significantly to an apparent
decline in cognitive ability.

Investigations
Patient's should be assessed using a standardised cognitive assessment tool.
To help identify reversible causes of dementia and exclude other causes of symptoms initial
investigations include:
In most cases:
Full blood count
Erythrocyte sedimentation rate (ESR)
Urea and electrolytes
Calcium
HbA1c
Liver function tests
Thyroid function tests
Serum B12 and folate levels
If clinically indicated:
A midstream urine, for example when delirium is a possibility
Chest X-ray, electrocardiogram (ECG), syphilis serology, and HIV testing
Patients with suspected dementia should be referred to a specialist memory assessment service
for further specialist assessment and management.
Specialist investigations for suspected dementia:
Should include structural imaging (MRI or CT scan) to: exclude non-dementia cerebral
pathology such as normal pressure hydrocephalus and to identify dementia subtype.
May also include:
HMPAO SPECT and FDG PET as second line investigations if diagnosis or subtype is
unclear
(FP-CIT) SPECT where a potential diagnosis of dementia with Lewy bodies (DLB) is in
doubt
Cerebrospinal fluid examination to exclude inflammatory, infective, or malignant
causes of dementia, in cases where dementia is rapidly progressive, the presentation
is unusual, or the person is younger than 55 years of age
Electroencephalography if a diagnosis of delirium, frontotemporal dementia, or
Creutzfeldt–Jakob disease is suspected, or in the assessment of associated seizure
disorder in people with dementia
Brain biopsy in highly selected people whose dementia is thought to be due to a
potentially reversible condition (such as cerebral vasculitis) that cannot be
diagnosed in another way

Management
Specialist non-pharmacological interventions for cognitive symptoms of dementia:
Structured group cognitive stimulation programs
Reminiscence therapy with discussion of past experiences
Tools such as life histories, shared memories, and familiar objects from the past
Specialist drug treatments for cognitive symptoms of dementia:
Acetylcholinesterase (AChE) inhibitors (donepezil, galantamine, and rivastigmine) - These
drugs can be used for mild to moderate Alzheimer's disease.
Memantine (a N-methyl-D-aspartic acid receptor antagonist) - Memantine is an option for
managing Alzheimer's disease for people with moderate Alzheimer's disease who are
intolerant of, or have a contraindication to, AChE inhibitors or in severe Alzheimer's disease.
Specialist drug treatment for behavioural and psychological symptoms of dementia (BPSD) or
non-cognitive symptoms of dementia and challenging behaviour:
No antipsychotics (except risperidone in very specific circumstances) are licensed for
BPSD in the UK. Specialists may prescribe an antipsychotic drug off-label in certain
circumstances (e.g. if they are severely distressed or there is an immediate risk of harm to
the person or others) following a thorough clinical assessment and discussion (including
the risks and benefits) with the carer and (if possible) the person with dementia.
Antipsychotics have potentially serious adverse effects, including increased risk of stroke
and mortality, Parkinsonism, and cognitive impairment. They should be used with caution,
at low dose, and for the shortest time possible.
In specific situations, specialists may offer AChE inhibitors for BPSD if non-
pharmacological methods and anti-psychotic drugs are inappropriate or ineffective.
Elderly Care & Fragility  QUESTION 8 OF 12

A 79 year old man is brought to the Emergency Department after being found
wandering the streets in the middle of the night by his neighbours. The neighbour
who accompanies him to the Emergency Department describes increasing confusion
over the last year and is concerned that he should not be living on his own. He has no
recorded next of kin and the neighbours are only aware of a daughter who lives
abroad. His GP record shows a diagnosis of Alzheimer's disease but you cannot see
any follow up. He keeps trying to leave the Emergency Department and becomes
distressed when you try to stop him. What is the most appropriate next step in this
patient's management?

A best-interests meeting should be arranged 6%

✓ A formal capacity assessment should be carried out 69%

He should be discharged home with a care package 2%

His daughter should be contacted 9%

x An independent mental capacity advocate (IMCA) should be appointed 14%

ANSWER
All of the options may well be appropriate; however, the most important first step would be to make an
assessment of capacity. The Mental Capacity Act 2005 aims to empower and protect people of 16 years of
age and over, who lack capacity to make certain decisions for themselves because of illness, learning
disabilities, or mental health problems. The five main principles of the Act are: there is a presumption of
capacity; the right for individuals to be supported to make their own decisions; it should not be assumed
that someone lacks capacity simply because their decisions might seem unwise or eccentric; if someone
lacks capacity, anything done on their behalf must be done in their best interests; and, if someone lacks
capacity, before making a decision on their behalf, all alternatives must be considered and the option
chosen should be the least restrictive of their basic rights and freedoms.
Dementia LAST UPDATED: 12TH
FEBRUARY 2021
ELDERLY CARE & FRAGILITY
 Bookmark

Dementia is a typically progressive clinical syndrome of deteriorating mental function significant


enough to interfere with activities of daily living (ADLs). It affects cognitive domains (such as memory,
thinking, language, orientation and judgement) and social behaviour (such as emotional control and
motivation).

Causes
Alzheimer’s disease (50 – 75% of cases)
Atrophy of the cerebral cortex
Formation of amyloid plaques and neurofibrillary tangles
Reduced neuronal acetylcholine production
Vascular dementia (up to 20% of cases)
Reduced blood supply to the brain
Dementia with Lewy bodies (DLB) (10 – 15% of cases)
Cortical and subcortical Lewy bodies (abnormal deposits of protein inside nerve cells)
Frontotemporal dementia (FTB) (2% of cases)
Progressive degeneration of the frontal and/or temporal lobe
Rarer causes of dementia:
Parkinson’s disease dementia
Progressive supranuclear palsy
Huntington’s disease
Prion disease (such as Creutzfeldt-Jakob disease [CJD])
Normal pressure hydrocephalus (NPH)
Chronic subdural haematoma
Benign tumours
Metabolic and endocrine disorders (such as chronic hypocalcaemia and recurrent
hypoglycaemia)
Vitamin deficiencies (such as B12 and thiamine deficiency)
Infections (such as HIV infection and syphilis)

Risk factors
Age - strongest risk factor for dementia
Mild cognitive impairment
Learning difficulties
Genetics
Cardiovascular disease risk factors (such as diabetes, smoking, hypercholesterolemia, and
hypertension)
Parkinson's disease
Stroke
Depression
Heavy alcohol consumption
Low educational attainment
Low social engagement and support

Clinical features
Dementia can be difficult to identify as it usually has an insidious onset and non-specific signs and
symptoms, which vary from person to person. People with early dementia may deny symptoms or
accommodate to cognitive change and functional ability.
Suspect dementia if any of the following are reported by the person and/or their family/carer:
Cognitive impairment, including:
Memory problems
Receptive or expressive dysphasia
Difficulty in carrying out coordinated movements such as dressing
Disorientation and unawareness of the time and place
Impairment of executive function, such as difficulties with planning and problem solving
Behavioural and psychological symptoms of dementia (BPSD) tend to fluctuate, may last for 6
months or more and include:
Psychosis
Agitation and emotional lability
Depression and anxiety
Withdrawal or apathy
Disinhibition
Motor disturbance
Sleep cycle disturbance or insomnia
Tendency to repeat phrases or questions
Difficulties with activities of daily living (ADLs):
In the early stages of dementia this may lead to neglect of household tasks, nutrition
(causing weight loss), personal hygiene, and grooming. People with dementia who are in
employment may find that they are increasingly making mistakes at work.
In the later stages, basic ADLs such as dressing, eating, and walking become affected.
Specific features of subtypes of dementia:
For Alzheimer’s disease:
Early impairment of episodic memory — this may include memory loss for recent events,
repeated questioning, and difficulty learning new information.
For vascular dementia:
Stepwise increases in the severity of symptoms — subcortical ischaemic vascular dementia
may present insidiously with gait and attention problems and changes in personality.
Focal neurological signs (such as hemiparesis or visual field defects) may be present.
For dementia with Lewy bodies:
Repeated falls, syncope or transient loss of consciousness, severe sensitivity to
antipsychotics, delusions, and hallucinations may be present. Memory impairment may not
be apparent in early stages. Parkinsonian motor features (such as shuffling gait, rigidity,
bradykinesia, and loss of spontaneous movement) and autonomic dysfunction (such as
postural hypotension, difficulty in swallowing, and incontinence or constipation) may be
present.
For frontotemporal dementia (FTD):
Personality change and behavioural disturbance (such as apathy or social/sexual
disinhibition) may develop insidiously. Other cognitive functions (such as memory and
perception) may be relatively preserved.

Differential diagnosis
Conditions that can present with similar symptoms to dementia include:
Normal-age related memory changes - Normal ageing is associated with a mild decline in
cognitive function, and memory lapses are common, especially during times of physical illness or
stress.
Mild cognitive impairment (MCI) - MCI differs from dementia in that symptoms do not fulfil the
diagnostic criteria for dementia, for example only one cognitive domain may be affected or
activities of daily life may not be significantly affected. Of people with MCI, 50% will later develop
dementia.
Depression - Symptoms of depression include low mood, loss of interest, anhedonia, and self-
neglect which can be similar to those of dementia.In older people, features of depression may be
less obvious, with somatic symptoms (such as reduced appetite, fatigue, and insomnia) are more
common.
Delirium - Delirium is an acute, fluctuating syndrome of disturbed consciousness, attention,
cognition, and perception. It is a common condition in the differential diagnosis for dementia.
People with cognitive impairment are at increased risk of delirium, and the two conditions often
coexist.
Vitamin deficiency - Thiamine deficiency can lead to Wernicke encephalopathy and Korsakoff
psychosis. Symptoms include confusion, memory loss, problems with learning new information
and gait disturbances. Vitamin B12 deficiency can lead to ataxia, psychiatric abnormalities,
memory loss, and gait disturbance.
Hypothyroidism - Symptoms of hypothyroidism can include low mood, and impaired
concentration and memory.
Adverse drug effects - Many drugs, including benzodiazepines, analgesics (such as opioids,
naproxen, and ibuprofen), anticholinergics, antidepressants (such as tricyclics), antipsychotics
(such as haloperidol), anticonvulsants (especially older preparations, such as phenytoin and
phenobarbital), and corticosteroids can affect cognition.
Normal pressure hydrocephalus - Normal pressure hydrocephalus can present with symptoms of
early cognitive impairment, urinary incontinence, and gait disorder. In NPH, cerebrospinal fluid
(CSF) flow has been impeded but compensatory mechanisms have prevented raised pressure.
The lateral ventricles are prominently dilated, and these ventricles exert local pressure on certain
brain areas to give the classical clinical picture. Pressure on the frontal lobes gives the dementia
and pressure on the medial side of the motor cortex, and the pyramidal tract fibres, cause
incontinence and pyramidal leg weakness. Diagnosis is by lumbar puncture (to demonstrate a
normal CSF opening pressure) followed by head computed tomography (CT)/magnetic resonance
imaging (MRI) (showing enlarged ventricles). Treatment is with ventriculoperitoneal shunting.
Sensory deficits - Problems with vision and hearing can contribute significantly to an apparent
decline in cognitive ability.

Investigations
Patient's should be assessed using a standardised cognitive assessment tool.
To help identify reversible causes of dementia and exclude other causes of symptoms initial
investigations include:
In most cases:
Full blood count
Erythrocyte sedimentation rate (ESR)
Urea and electrolytes
Calcium
HbA1c
Liver function tests
Thyroid function tests
Serum B12 and folate levels
If clinically indicated:
A midstream urine, for example when delirium is a possibility
Chest X-ray, electrocardiogram (ECG), syphilis serology, and HIV testing
Patients with suspected dementia should be referred to a specialist memory assessment service
for further specialist assessment and management.
Specialist investigations for suspected dementia:
Should include structural imaging (MRI or CT scan) to: exclude non-dementia cerebral
pathology such as normal pressure hydrocephalus and to identify dementia subtype.
May also include:
HMPAO SPECT and FDG PET as second line investigations if diagnosis or subtype is
unclear
(FP-CIT) SPECT where a potential diagnosis of dementia with Lewy bodies (DLB) is in
doubt
Cerebrospinal fluid examination to exclude inflammatory, infective, or malignant
causes of dementia, in cases where dementia is rapidly progressive, the presentation
is unusual, or the person is younger than 55 years of age
Electroencephalography if a diagnosis of delirium, frontotemporal dementia, or
Creutzfeldt–Jakob disease is suspected, or in the assessment of associated seizure
disorder in people with dementia
Brain biopsy in highly selected people whose dementia is thought to be due to a
potentially reversible condition (such as cerebral vasculitis) that cannot be
diagnosed in another way

Management
Specialist non-pharmacological interventions for cognitive symptoms of dementia:
Structured group cognitive stimulation programs
Reminiscence therapy with discussion of past experiences
Tools such as life histories, shared memories, and familiar objects from the past
Specialist drug treatments for cognitive symptoms of dementia:
Acetylcholinesterase (AChE) inhibitors (donepezil, galantamine, and rivastigmine) - These
drugs can be used for mild to moderate Alzheimer's disease.
Memantine (a N-methyl-D-aspartic acid receptor antagonist) - Memantine is an option for
managing Alzheimer's disease for people with moderate Alzheimer's disease who are
intolerant of, or have a contraindication to, AChE inhibitors or in severe Alzheimer's disease.
Specialist drug treatment for behavioural and psychological symptoms of dementia (BPSD) or
non-cognitive symptoms of dementia and challenging behaviour:
No antipsychotics (except risperidone in very specific circumstances) are licensed for
BPSD in the UK. Specialists may prescribe an antipsychotic drug off-label in certain
circumstances (e.g. if they are severely distressed or there is an immediate risk of harm to
the person or others) following a thorough clinical assessment and discussion (including
the risks and benefits) with the carer and (if possible) the person with dementia.
Antipsychotics have potentially serious adverse effects, including increased risk of stroke
and mortality, Parkinsonism, and cognitive impairment. They should be used with caution,
at low dose, and for the shortest time possible.
In specific situations, specialists may offer AChE inhibitors for BPSD if non-
pharmacological methods and anti-psychotic drugs are inappropriate or ineffective.
Elderly Care & Fragility  QUESTION 9 OF 12

A 76 year old woman presents to the Emergency Department with a 2 day history of
lower back pain. The pain is worse on movement. She denies trauma and tells you the
pain started suddenly 2 days ago whilst gardening. She has been taking regular
paracetamol and ibuprofen but tells you the pain is no better. She has a past medical
history of polymyalgia rheumatica. What is the diagnosis?

Intervertebral disc prolapse 6%

✓ Vertebral fracture 84%

Paget’s disease 3%

Ankylosing spondylitis 5%

x Vertebral osteomyelitis 2%
ANSWER
There is an L4 compression fracture most likely secondary to osteoporosis. Vertebral compression
fractures are the most common type of osteoporotic fracture. They often occur at the midthoracic (T7 -
T8) spine and the thoracolumbar junction (T12 - L1). Fractures may result in significant back pain, limited
physical functioning and activities of daily living, and can lead to loss of independence, depression, and
chronic pain.

Osteoporosis and Fragility Fractures LAST UPDATED: 5TH MARCH


2021
ELDERLY CARE & FRAGILITY
 Bookmark

Osteoporosis is a disease characterised by low bone mass and structural deterioration of bone tissue,
with a consequent increase in bone fragility and susceptibility to fracture. Osteoporosis is the end
result of an imbalance in the normal process of bone remodelling by osteoclasts and osteoblasts. It is
asymptomatic and often remains undiagnosed until a fragility fracture occurs.

Definitions
Osteoporosis is defined by the World Health Organization as a bone mineral density (BMD) of 2.5
standard deviations below the mean peak mass (average of young healthy adults) as measured
by dual-energy X-ray absorptiometry (DXA) applied to the femoral neck and reported as a T-
score.
An osteoporotic fracture is a fragility fracture occurring as a consequence of osteoporosis.
Osteoporotic fractures characteristically occur in the wrist, spine and hip, but can also occur in
the arm, pelvis, ribs and other bones.
A fragility fracture is defined as a fracture following a fall from standing height or less, although
vertebral fractures may occur spontaneously, or as a result of routine activities such as bending
or lifting.

Risk factors
The risk of getting an osteoporotic fracture depends on the person's risk of falls, their bone strength
(determined by bone mineral density), and other risk factors.
Risk factors for having a fragility fracture:
Risks factors affecting bone strength that reduce BMD:
Endocrine disease e.g. diabetes mellitus, hyperthyroidism, hyperparathyroidism
Malabsorptive disease e.g. IBD, coeliac disease, chronic pancreatitis
Chronic kidney disease
Chronic liver disease
COPD
Menopause
Immobility
BMI < 18.5 kg/m2
Risks factors affecting bone strength that do not reduce BMD:
Age
Oral corticosteroids
Smoking
Alcohol
Previous fragility fracture
Rheumatological disease e.g. rheumatoid arthritis
Parental history of hip fracture
Risk factors affecting bone strength with unestablished mechanisms:
Selective serotonin reuptake inhibitors (SSRIs)
Proton pump inhibitors (PPI)
Anticonvulsant drugs e.g. carbamazepine
Risk factors for falls:
Impaired vision
Neuromuscular weakness and incoordination
Cognitive impairment
Use of alcohol and sedative drugs

Assessment for fragility fracture risk


Exclude non-osteoporotic causes for fragility fractures:
Metastatic bone disease (bone pain, history of cancer (especially lung, thyroid, prostate,
kidney, or breast cancer), or symptoms of undiagnosed cancer (for example unexplained
general malaise or weight loss)
Multiple myeloma (bone pain, anaemia, recurrent infection, bleeding, hypercalcaemia,
kidney disease)
Osteomalacia (bone pain, muscle pain or proximal muscle weakness)
Paget’s disease (bone pain or deformity)
Aneurysmal bone cyst or hemangioma
Infection e.g. osteomyelitis, Pott's disease
Exclude secondary causes of osteoporosis especially in people with a fragility fracture who are at
low risk, including men of any age, pre-menopausal women, and women with premature
menopause (under 40 years of age)
Consider starting drug treatment in people with vertebral or hip fractures without undertaking
DXA if this is considered inappropriate or impractical
Offer a dual-energy X-ray absorptiometry (DXA) scan to measure bone mineral density (BMD)
without calculating the fragility fracture risk in people:
Over 50 years of age with a history of fragility fracture
Younger than 40 years of age who have a major risk factor for fragility fracture
For all other people with risk factors for osteoporosis, calculate the 10-year fragility fracture risk
prior to arranging a DXA scan to measure BMD; consider using the online risk calculators
QFracture® (preferred) or FRAX®, which predict the absolute risk of hip fracture and major
osteoporotic fractures (spine, wrist, hip, or shoulder) over 10 years
High risk: For people whose fracture risk is above the recommended threshold, offer a dual-
energy x-ray absorptiometry (DXA) scan, then bone-sparing drug treatment if the T-score
is -2.5 or less; if the T-score is greater than -2.5, modify risk factors where possible, treat
any underlying conditions, and repeat the DXA at an interval appropriate for the person
based on their risk profile, using clinical judgement (but usually within 2 years)
Intermediate risk: For people whose fracture risk is close to the recommended threshold
and who have risk factors that may be underestimated by FRAX®, arrange a DXA scan to
measure their bone mineral density (BMD) and offer drug treatment if the T-score is -2.5 or
less
Low risk: For people whose fracture risk is below the recommended threshold, do not offer
drug treatment, offer lifestyle advice and follow up within 5 years
Assess for vitamin D deficiency and inadequate calcium intake.
People are at risk of vitamin D deficiency if they are aged over 65 years, or are not exposed
to much sunlight (because they are confined indoors for long periods, or because they wear
clothes that cover the whole body)
A calcium intake of at least 1000 mg/day is recommended for people at increased risk of a
fragility fracture
Identify any risk factors for falls

Management
Lifestyle advice
Advise the person to:
Take regular exercise to improve muscle strength
Eat a balanced diet
Stop smoking if needed
Drink alcohol within recommended limits
Bone-sparing treatment
Prescribe a bisphosphonate (alendronate 10 mg once daily or 70 mg once weekly, or
risedronate 5 mg once daily or 35 mg once weekly), if there are no contraindications and
after appropriate counselling, to postmenopausal women and men over 50 years of age
who have been confirmed by dual-energy X-ray absorptiometry (DXA) scan to have
osteoporosis (bone mineral density [BMD] T-score of -2.5 or less).
Consider prescribing to people who are taking high doses of oral corticosteroids (more than
or equivalent to prednisolone 7.5 mg daily for 3 months or longer).
If an oral bisphosphonate is not tolerated or is contraindicated, consider specialist referral.
Specialist treatment options include zoledronic acid, strontium ranelate, raloxifene,
denosumab, and teriparatide.
Calcium and vitamin D supplements
If the person's calcium intake is adequate (700 mg/day), prescribe 10 micrograms (400
international units) of vitamin D (without calcium) for people not exposed to much sunlight.
If calcium intake is inadequate prescribe 10 micrograms (400 international units) of vitamin
D with at least 1000 mg of calcium daily; prescribe 20 micrograms (800 international units)
of vitamin D with at least 1000 mg of calcium daily for elderly people who are housebound
or living in a nursing home.
Hormone replacement therapy
Consider prescribing hormone replacement therapy (HRT) to women who have a premature
menopause (menopause before 40 years of age) to reduce the risk of fragility fractures and
for the relief of menopausal symptoms.

Vertebral compression fractures


Vertebral compression fractures are the most common type of osteoporotic fracture. They often occur
at the midthoracic (T7 - T8) spine and the thoracolumbar junction (T12 - L1).
Fractures may result in significant back pain, limited physical functioning and activities of daily living,
and can lead to loss of independence, depression, and chronic pain. Osteoporotic vertebral
compression that occurs slowly over time is often asymptomatic. Old or healed fractures may be an
incidental finding on radiographs of the chest or abdomen. In other patients, the presence of vertebral
fractures may become apparent because of height loss or kyphosis. In contrast, acute episodes of
significant vertebral body compression are associated with pain.
In patients who have acute symptomatic vertebral body fracture, there is often no history of preceding
trauma. The typical patient presents with acute back pain after sudden bending, coughing, or
lifting. The pain is usually well localised to the midline spine but often refers in a unilateral or bilateral
pattern into the flank, anterior abdomen, or the posterior superior iliac spine. On physical examination,
the patient may experience pain upon palpation and percussion of the corresponding spinous process
and paravertebral structures.
Acute episodes of pain following a vertebral body fracture usually resolve after four to six weeks, but
pain may persist for longer periods (many months), indicating an unhealed or slowly healing
fracture. The initial management of osteoporotic vertebral compression fractures should include pain
control and activity modification (bed rest is not recommended). The indications for and timing of
vertebral augmentation procedures are controversial. Most patients with vertebral compression
fractures can be treated successfully with conservative management.
Elderly Care & Fragility  QUESTION 10 OF 12

A 79 year old man is brought to the Emergency Department by his daughter. She is
struggling to care for him at home due to episodes of cognitive impairment and
memory loss. She describes days when he seems to be his normal self, interspersed
with days of confusion and memory loss. She has occasionally found him appearing
to converse with an unseen individual. He has a past medical history of COPD and
type 2 diabetes. The only finding of note in his neurological examination is increased
tone in the right arm. What is the most likely diagnosis?

Depressive pseudodementia 2%

Frontotemporal dementia 12%

Korsakoff psychosis 4%

✓ Lewy body dementia 67%

x Vascular dementia 15%

ANSWER
Characteristic features of Lewy body dementia include day-to-day fluctuating levels of cognitive
functioning, visual hallucinations, sleep disturbance, transient loss of consciousness, recurrent falls and
Parkinsonian features (tremor, hypokinesia, rigidity and postural instability). Although people with Lewy
body dementia are prone to hallucination, antipsychotics should be avoided as they precipitate severe
Parkinsonism in 60%. A Lewy body is an abnormality of the cytoplasm found within a neurone, containing
various proteins and granular material. They are found in the cerebral cortex in patients with Lewy body
dementia, and they are also found in patients with Parkinson’s disease.

Dementia LAST UPDATED: 12TH


FEBRUARY 2021
ELDERLY CARE & FRAGILITY
 Bookmark

Dementia is a typically progressive clinical syndrome of deteriorating mental function significant


enough to interfere with activities of daily living (ADLs). It affects cognitive domains (such as memory,
thinking, language, orientation and judgement) and social behaviour (such as emotional control and
motivation).

Causes
Alzheimer’s disease (50 – 75% of cases)
Atrophy of the cerebral cortex
Formation of amyloid plaques and neurofibrillary tangles
Reduced neuronal acetylcholine production
Vascular dementia (up to 20% of cases)
Reduced blood supply to the brain
Dementia with Lewy bodies (DLB) (10 – 15% of cases)
Cortical and subcortical Lewy bodies (abnormal deposits of protein inside nerve cells)
Frontotemporal dementia (FTB) (2% of cases)
Progressive degeneration of the frontal and/or temporal lobe
Rarer causes of dementia:
Parkinson’s disease dementia
Progressive supranuclear palsy
Huntington’s disease
Prion disease (such as Creutzfeldt-Jakob disease [CJD])
Normal pressure hydrocephalus (NPH)
Chronic subdural haematoma
Benign tumours
Metabolic and endocrine disorders (such as chronic hypocalcaemia and recurrent
hypoglycaemia)
Vitamin deficiencies (such as B12 and thiamine deficiency)
Infections (such as HIV infection and syphilis)

Risk factors
Age - strongest risk factor for dementia
Mild cognitive impairment
Learning difficulties
Genetics
Cardiovascular disease risk factors (such as diabetes, smoking, hypercholesterolemia, and
hypertension)
Parkinson's disease
Stroke
Depression
Heavy alcohol consumption
Low educational attainment
Low social engagement and support

Clinical features
Dementia can be difficult to identify as it usually has an insidious onset and non-specific signs and
symptoms, which vary from person to person. People with early dementia may deny symptoms or
accommodate to cognitive change and functional ability.
Suspect dementia if any of the following are reported by the person and/or their family/carer:
Cognitive impairment, including:
Memory problems
Receptive or expressive dysphasia
Difficulty in carrying out coordinated movements such as dressing
Disorientation and unawareness of the time and place
Impairment of executive function, such as difficulties with planning and problem solving
Behavioural and psychological symptoms of dementia (BPSD) tend to fluctuate, may last for 6
months or more and include:
Psychosis
Agitation and emotional lability
Depression and anxiety
Withdrawal or apathy
Disinhibition
Motor disturbance
Sleep cycle disturbance or insomnia
Tendency to repeat phrases or questions
Difficulties with activities of daily living (ADLs):
In the early stages of dementia this may lead to neglect of household tasks, nutrition
(causing weight loss), personal hygiene, and grooming. People with dementia who are in
employment may find that they are increasingly making mistakes at work.
In the later stages, basic ADLs such as dressing, eating, and walking become affected.
Specific features of subtypes of dementia:
For Alzheimer’s disease:
Early impairment of episodic memory — this may include memory loss for recent events,
repeated questioning, and difficulty learning new information.
For vascular dementia:
Stepwise increases in the severity of symptoms — subcortical ischaemic vascular dementia
may present insidiously with gait and attention problems and changes in personality.
Focal neurological signs (such as hemiparesis or visual field defects) may be present.
For dementia with Lewy bodies:
Repeated falls, syncope or transient loss of consciousness, severe sensitivity to
antipsychotics, delusions, and hallucinations may be present. Memory impairment may not
be apparent in early stages. Parkinsonian motor features (such as shuffling gait, rigidity,
bradykinesia, and loss of spontaneous movement) and autonomic dysfunction (such as
postural hypotension, difficulty in swallowing, and incontinence or constipation) may be
present.
For frontotemporal dementia (FTD):
Personality change and behavioural disturbance (such as apathy or social/sexual
disinhibition) may develop insidiously. Other cognitive functions (such as memory and
perception) may be relatively preserved.

Differential diagnosis
Conditions that can present with similar symptoms to dementia include:
Normal-age related memory changes - Normal ageing is associated with a mild decline in
cognitive function, and memory lapses are common, especially during times of physical illness or
stress.
Mild cognitive impairment (MCI) - MCI differs from dementia in that symptoms do not fulfil the
diagnostic criteria for dementia, for example only one cognitive domain may be affected or
activities of daily life may not be significantly affected. Of people with MCI, 50% will later develop
dementia.
Depression - Symptoms of depression include low mood, loss of interest, anhedonia, and self-
neglect which can be similar to those of dementia.In older people, features of depression may be
less obvious, with somatic symptoms (such as reduced appetite, fatigue, and insomnia) are more
common.
Delirium - Delirium is an acute, fluctuating syndrome of disturbed consciousness, attention,
cognition, and perception. It is a common condition in the differential diagnosis for dementia.
People with cognitive impairment are at increased risk of delirium, and the two conditions often
coexist.
Vitamin deficiency - Thiamine deficiency can lead to Wernicke encephalopathy and Korsakoff
psychosis. Symptoms include confusion, memory loss, problems with learning new information
and gait disturbances. Vitamin B12 deficiency can lead to ataxia, psychiatric abnormalities,
memory loss, and gait disturbance.
Hypothyroidism - Symptoms of hypothyroidism can include low mood, and impaired
concentration and memory.
Adverse drug effects - Many drugs, including benzodiazepines, analgesics (such as opioids,
naproxen, and ibuprofen), anticholinergics, antidepressants (such as tricyclics), antipsychotics
(such as haloperidol), anticonvulsants (especially older preparations, such as phenytoin and
phenobarbital), and corticosteroids can affect cognition.
Normal pressure hydrocephalus - Normal pressure hydrocephalus can present with symptoms of
early cognitive impairment, urinary incontinence, and gait disorder. In NPH, cerebrospinal fluid
(CSF) flow has been impeded but compensatory mechanisms have prevented raised pressure.
The lateral ventricles are prominently dilated, and these ventricles exert local pressure on certain
brain areas to give the classical clinical picture. Pressure on the frontal lobes gives the dementia
and pressure on the medial side of the motor cortex, and the pyramidal tract fibres, cause
incontinence and pyramidal leg weakness. Diagnosis is by lumbar puncture (to demonstrate a
normal CSF opening pressure) followed by head computed tomography (CT)/magnetic resonance
imaging (MRI) (showing enlarged ventricles). Treatment is with ventriculoperitoneal shunting.
Sensory deficits - Problems with vision and hearing can contribute significantly to an apparent
decline in cognitive ability.

Investigations
Patient's should be assessed using a standardised cognitive assessment tool.
To help identify reversible causes of dementia and exclude other causes of symptoms initial
investigations include:
In most cases:
Full blood count
Erythrocyte sedimentation rate (ESR)
Urea and electrolytes
Calcium
HbA1c
Liver function tests
Thyroid function tests
Serum B12 and folate levels
If clinically indicated:
A midstream urine, for example when delirium is a possibility
Chest X-ray, electrocardiogram (ECG), syphilis serology, and HIV testing
Patients with suspected dementia should be referred to a specialist memory assessment service
for further specialist assessment and management.
Specialist investigations for suspected dementia:
Should include structural imaging (MRI or CT scan) to: exclude non-dementia cerebral
pathology such as normal pressure hydrocephalus and to identify dementia subtype.
May also include:
HMPAO SPECT and FDG PET as second line investigations if diagnosis or subtype is
unclear
(FP-CIT) SPECT where a potential diagnosis of dementia with Lewy bodies (DLB) is in
doubt
Cerebrospinal fluid examination to exclude inflammatory, infective, or malignant
causes of dementia, in cases where dementia is rapidly progressive, the presentation
is unusual, or the person is younger than 55 years of age
Electroencephalography if a diagnosis of delirium, frontotemporal dementia, or
Creutzfeldt–Jakob disease is suspected, or in the assessment of associated seizure
disorder in people with dementia
Brain biopsy in highly selected people whose dementia is thought to be due to a
potentially reversible condition (such as cerebral vasculitis) that cannot be
diagnosed in another way

Management
Specialist non-pharmacological interventions for cognitive symptoms of dementia:
Structured group cognitive stimulation programs
Reminiscence therapy with discussion of past experiences
Tools such as life histories, shared memories, and familiar objects from the past
Specialist drug treatments for cognitive symptoms of dementia:
Acetylcholinesterase (AChE) inhibitors (donepezil, galantamine, and rivastigmine) - These
drugs can be used for mild to moderate Alzheimer's disease.
Memantine (a N-methyl-D-aspartic acid receptor antagonist) - Memantine is an option for
managing Alzheimer's disease for people with moderate Alzheimer's disease who are
intolerant of, or have a contraindication to, AChE inhibitors or in severe Alzheimer's disease.
Specialist drug treatment for behavioural and psychological symptoms of dementia (BPSD) or
non-cognitive symptoms of dementia and challenging behaviour:
No antipsychotics (except risperidone in very specific circumstances) are licensed for
BPSD in the UK. Specialists may prescribe an antipsychotic drug off-label in certain
circumstances (e.g. if they are severely distressed or there is an immediate risk of harm to
the person or others) following a thorough clinical assessment and discussion (including
the risks and benefits) with the carer and (if possible) the person with dementia.
Antipsychotics have potentially serious adverse effects, including increased risk of stroke
and mortality, Parkinsonism, and cognitive impairment. They should be used with caution,
at low dose, and for the shortest time possible.
In specific situations, specialists may offer AChE inhibitors for BPSD if non-
pharmacological methods and anti-psychotic drugs are inappropriate or ineffective.
Elderly Care & Fragility  QUESTION 11 OF 12

An 83 year old man is brought to the Emergency Department by your local police
service. They tell you he was found in a nearby supermarket causing a disturbance.
His medical records show he has been referred to the local memory clinic by his GP
but has not yet been seen. The triage nurse is finding it difficult to obtain
observations as the patient is so distressed. She is able to record his temperature as
38.5°C. He is distressed and is trying to leave the department. You assess his
capacity to decline assessment and treatment and you feel he is lacking capacity for
this decision at this time. What is the next management step for this patient?

✓ 1 mg haloperidol intramuscularly 44%

10 mg haloperidol intramuscularly 16%

1 mg lorazepam intramuscularly 30%

2 mg lorazepam intravenously 7%

x 2.5 mg olanzapine orally 2%

ANSWER
This man is likely to be suffering from delirium, probably on the background of dementia. He needs urgent
investigation and treatment but is currently a risk to himself. NICE guidance for management of delirium
only recommends pharmacological treatment if a patient presents a danger to himself or others, and
verbal and non-verbal de-escalation techniques are ineffective or inappropriate. It is unlikely that this
patient will take any oral medication. Low-dose haloperidol is the first line drug. Start at the lowest
clinically appropriate dose and titrate cautiously according to symptoms.

Delirium LAST UPDATED: 12TH


FEBRUARY 2021
ELDERLY CARE & FRAGILITY
 Bookmark

Delirium (sometimes called 'acute confusional state') is an acute, fluctuating syndrome of disturbed
consciousness, attention, cognition and perception.
Delirium usually develops acutely over hours to days and behavioural disturbance, personality changes,
and psychotic features may occur. Symptoms typically fluctuate (come and go or increase and
decrease in severity). Lucid intervals usually occur during the day with the worst disturbance at night.
Delirium typically occurs in people with predisposing factors (such as advanced age or multiple
comorbidities) when new precipitating factors (such as some medications or infection) are added.

Predisposing factors
Predisposing factors include:
Older age (over 65 years)
Cognitive impairment (such as dementia)
Frailty/multiple comorbidities (such as stroke or heart failure)
Significant injuries such as hip fracture
Functional impairment (for example immobility or the use of physical restraints such as cot sides)
Iatrogenic events (such as bladder catheterisation, polypharmacy, or surgery)
History of, or current, alcohol excess
Sensory impairment (such as visual impairment or hearing loss)
Poor nutrition
Lack of stimulation
Terminal phase of illness

Precipitating factors
Precipitating factors include:
Infection such as urinary tract infection, infected pressure sore, or pneumonia
Metabolic disturbance such as hypoglycaemia, hyperglycaemia, or electrolyte abnormalities
Cardiovascular disorders such as myocardial infarction or heart failure
Respiratory disorders such as pulmonary embolism or exacerbation of chronic obstructive
pulmonary disease
Neurological disorders such as stroke, encephalitis, or subdural haematoma
Endocrine disorders such as thyroid dysfunction or Cushing's syndrome
Urological disorders such as urinary retention
Gastrointestinal disorders such as hepatic failure, constipation (including faecal impaction), or
malnutrition
Severe uncontrolled pain
Alcohol intoxication or withdrawal
Medication e.g. opioids, benzodiazepines (use and withdrawal), dihydropyridines (such as
amlodipine), histamine-2 receptor antagonists, anti-Parkinsonian medications, tricyclic
antidepressants, lithium, antipsychotics, anticonvulsants, antiarrhythmics, antihypertensives,
histamine-2 receptor antagonists, corticosteroids, and NSAIDs
Psychosocial factors such as depression, sleep deprivation, visual or hearing impairment,
emotional stress, or change of environment

Complications
The complications of delirium include:
Increased mortality
Increased length of stay in hospital
Nosocomial infections
Increased risk of admission to long-term care or re-admission to hospital
Increased incidence of dementia
Falls
Pressure sores
Continence problems
Malnutrition
Functional impairment
Distress for the person, their family, and/or carers

Clinical features
Suspect delirium in people with a sudden change in behaviour that may be reported by the person, a
carer, or relative. Behavioural changes may include:
Altered cognitive function — the person may be disoriented, have memory and language
impairment, worsened concentration, slow responses, and confusion. The person may not be
able to recall details of their current illness, instructions, or names.
Inattention — the person may be easily distractible and have difficulty focusing and moving
attention from one thing to another, for example they are unable to maintain a conversation or
follow reasonable commands.
Disorganised thinking — the person may have disorganised, rambling, or irrelevant conversation,
unclear or illogical flow of ideas, and difficulty expressing their needs and concerns.
Altered perception — the person may experience paranoid delusions, misperceptions or, visual or
auditory hallucinations which may be distressing.
Altered physical function:
Hyperactive delirium — the person may have increased sensitivity to their immediate
surroundings with agitation, restlessness, sleep disturbance, and hypervigilance.
Restlessness and wandering are common.
Hypoactive delirium (more common) — the person may be lethargic, have reduced mobility
and movement, lack interest in daily activities, have a reduced appetite, and become quiet
and withdrawn.
Mixed — the person will have a combination of signs and symptoms of hyperactive and
hypoactive subtypes.
Altered social behaviour — the person may have intermittent and labile changes in mood and/or
emotions (such as fear, paranoia, anxiety, depression, irritability, apathy, anger, or euphoria). Their
behaviour may be inappropriate and they may not cooperate with reasonable requests or become
withdrawn.
Altered level of consciousness — the person may have a clouding of consciousness, reduced
awareness of their surroundings, and sleep-cycle disturbances (such as daytime drowsiness,
night-time insomnia, disturbed sleep, or complete sleep cycle reversal). Impaired consciousness
can be subtle, and may initially only be apparent as lethargy or distractibility.

Differential diagnosis
Depression - Depression can present with similar symptoms to delirium (especially hypoactive
delirium), for example mood change, anorexia, sleep disturbance, and psychomotor change.
Dementia - Apart from Lewy-body dementia (which has fluctuation in cognition as one of its
features), dementia usually has an insidious onset and is not associated with fluctuations in
mental state.
Mental illness - Late-onset mania or schizophrenia can present with features similar to
hyperactive delirium.
Anxiety - Behavioural changes associated with delirium can include anxiety.
Thyroid disease - Hyperthyroidism and hypothyroidism can have similar features to hyperactive
delirium and hypoactive delirium respectively.
Non-convulsive epilepsy or temporal lobe epilepsy - Non-convulsive epilepsy can present with
subtle behaviour or mood changes and clouding of consciousness.
Charles Bonnet syndrome - Visual hallucinations may occur in people with severe visual
impairment and can range from simple patterns of straight lines to detailed pictures of people or
buildings.

Assessment
Delirium is a clinical diagnosis based on a detailed history, examination, and relevant investigations.
Take a history from the person and an informed observer (family member or carer).
Check vital signs including temperature, blood pressure, heart rate, capillary refill time, finger-
prick blood glucose, and pulse oximetry — to identify fever, hypoperfusion, hyperglycaemia,
hypoglycaemia, or hypoxia.
Carry out a general examination to identify precipitating factors.
Confirm a diagnosis of delirium by carrying out a cognitive assessment.
Arrange targeted investigations based on findings from the history and examination, for example:
Urinalysis — to identify conditions such as infection or hyperglycaemia. Arrange a mid-
stream urine (MSU) if urinalysis is abnormal.
Sputum culture — to identify chest infection.
Full blood count — to identify infection or anaemia.
Folate and B12 — to identify vitamin deficiency.
Urea and electrolytes — to identify acute kidney injury and electrolyte disturbance (such as
hyponatraemia or hypokalaemia).
HbA1c — to identify hyperglycaemia.
Calcium — to identify hypercalcaemia or hypocalcaemia.
Liver function tests — to identify hepatic failure and rule out hepatic encephalopathy.
Inflammatory markers (such as erythrocyte sedimentation rate and C-reactive protein) —
these tests are non-discriminatory but can help to identify infection or inflammation.
Drug levels — to identify drug toxicity, for example if the person has taken digoxin, lithium,
or alcohol.
Thyroid function tests — to identify hyperthyroidism or hypothyroidism.
Chest X-ray — to identify conditions such as pneumonia and heart failure.
Electrocardiogram — to identify cardiac conditions including arrhythmias.

Management
Most people with delirium should be admitted to hospital for urgent assessment, close
monitoring, and treatment. The decision as to whether to admit a person with delirium depends
on the person's specific clinical and social situation, and should also take into account the views
of family members or carers. If the person is deemed to not have capacity to consent, decisions
should be made in the best interests of the person using the Mental Capacity Act 2005. If the
person with delirium refuses admission ask carers or family (if appropriate) to help persuade the
person. If this fails, consider admission under the Mental Capacity Act (2005).
Management of a person with delirium includes:
Correcting any precipitating factors e.g. infection, drugs, constipation, urinary retention,
dehydration and electrolyte imbalance, pain
Optimising treatment of comorbidities
Managing behaviour change
Trying reorientation strategies e.g. regular cues, continuity of care from carers and
staff
Maintaining safe mobility e.g. avoiding physical restraint, encouraging walking
Normalising the sleep-wake cycle e.g. discouraging napping and encouraging
uninterrupted sleep at night
Managing challenging behaviour (such as aggression, agitation or shouting)
Addressing any underlying causes for the behaviour (such as discomfort, thirst, or
need for the toilet).
Moving the person to a safe, low-stimulation environment (such as a quiet room).
Using verbal and non-verbal de-escalation techniques (such as active listening,
effective verbal responding, pictures, and symbols).
Pharmacological measures
Specialists may suggest pharmacological measures as a last resort for severe
agitation or psychosis if:
Verbal and non-verbal de-escalation techniques are inappropriate or have
failed, and
The person is a danger to themselves or others, and
The cause of delirium is known and being treated, and
The benefit outweighs the risk to the person, and
There is enough care in place for the person to be continually monitored.
The following medication may be suggested:
Short-term (for 1 week or less) low-dose haloperidol (off-label indication).
Low-dose lorazepam (off-label indication) as an alternative if haloperidol is
contraindicated (for example in people with Parkinson's disease/parkinsonism,
Lewy-body dementia, or a prolonged QT interval).
Start at the lowest clinically appropriate dose and titrate cautiously according to
symptoms. The aim of drug treatment is to calm (not sedate) the person.
Elderly Care & Fragility  QUESTION 12 OF 12

A 79 year old man is brought to the Emergency Department by his daughter. She is
concerned about his cognition and memory, she brought him today because she is
concerned for his safety at home. She tell you that sometimes he is "completely
normal" but on other days he is "muddled" and cannot remember anything. The
patient is unsure why he is in the Emergency Department but does tell you he often
sees faces when there are none. He has no significant past medical history. On
examination you note a rigid tone in the right arm and a slow tremor of the left hand.
The rest of his physical examination is unremarkable. What is the likely diagnosis?

Depressive pseudodementia 2%

Frontotemporal dementia 10%

Korsakoff psychosis 4%

✓ Lewy body dementia 79%

x Vascular dementia 6%

ANSWER

Frontotemporal dementia is very rare, and usually presents with personality change and
disinhibition with a sparing of memory function early on in the disease.
Vascular dementia occurs with multiple transient ischaemic attacks (TIAs) and strokes, and while
this patient is a smoker, there is no history of strokes or TIAs, and no neurological deficit on
examination, suggestive of a stroke.
Korsakoff psychosis is characterised by severe impairment of episodic memory function – patients
are unable to form new episodic memories, and have impaired access to old episodic memories, and
therefore are liable to confabulate. It occurs normally secondary to untreated Wernicke
encephalopathy, although it can be secondary to hypoxic brain damage or CNS infection.
Depressive pseudodementia presents similarly to Alzheimer’s disease, and is a common cause of
misdiagnosis. It features depressed mood and anxiety, and lack of self-confidence can lead to an
apparent short-term memory deficit, although with encouragement and close examination the
memory deficit disappears.
Lewy body dementia is the second most common dementia after Alzheimer’s disease. Characteristic
features of Lewy body dementia include day-today fluctuating levels of cognitive functioning, visual
hallucinations, sleep disturbance, transient loss of consciousness, recurrent falls and parkinsonian
features (tremor, hypokinesia, rigidity and postural instability).

Dementia LAST UPDATED: 12TH


FEBRUARY 2021
ELDERLY CARE & FRAGILITY
 Bookmark

Dementia is a typically progressive clinical syndrome of deteriorating mental function significant


enough to interfere with activities of daily living (ADLs). It affects cognitive domains (such as memory,
thinking, language, orientation and judgement) and social behaviour (such as emotional control and
motivation).

Causes
Alzheimer’s disease (50 – 75% of cases)
Atrophy of the cerebral cortex
Formation of amyloid plaques and neurofibrillary tangles
Reduced neuronal acetylcholine production
Vascular dementia (up to 20% of cases)
Reduced blood supply to the brain
Dementia with Lewy bodies (DLB) (10 – 15% of cases)
Cortical and subcortical Lewy bodies (abnormal deposits of protein inside nerve cells)
Frontotemporal dementia (FTB) (2% of cases)
Progressive degeneration of the frontal and/or temporal lobe
Rarer causes of dementia:
Parkinson’s disease dementia
Progressive supranuclear palsy
Huntington’s disease
Prion disease (such as Creutzfeldt-Jakob disease [CJD])
Normal pressure hydrocephalus (NPH)
Chronic subdural haematoma
Benign tumours
Metabolic and endocrine disorders (such as chronic hypocalcaemia and recurrent
hypoglycaemia)
Vitamin deficiencies (such as B12 and thiamine deficiency)
Infections (such as HIV infection and syphilis)
Risk factors
Age - strongest risk factor for dementia
Mild cognitive impairment
Learning difficulties
Genetics
Cardiovascular disease risk factors (such as diabetes, smoking, hypercholesterolemia, and
hypertension)
Parkinson's disease
Stroke
Depression
Heavy alcohol consumption
Low educational attainment
Low social engagement and support

Clinical features
Dementia can be difficult to identify as it usually has an insidious onset and non-specific signs and
symptoms, which vary from person to person. People with early dementia may deny symptoms or
accommodate to cognitive change and functional ability.
Suspect dementia if any of the following are reported by the person and/or their family/carer:
Cognitive impairment, including:
Memory problems
Receptive or expressive dysphasia
Difficulty in carrying out coordinated movements such as dressing
Disorientation and unawareness of the time and place
Impairment of executive function, such as difficulties with planning and problem solving
Behavioural and psychological symptoms of dementia (BPSD) tend to fluctuate, may last for 6
months or more and include:
Psychosis
Agitation and emotional lability
Depression and anxiety
Withdrawal or apathy
Disinhibition
Motor disturbance
Sleep cycle disturbance or insomnia
Tendency to repeat phrases or questions
Difficulties with activities of daily living (ADLs):
In the early stages of dementia this may lead to neglect of household tasks, nutrition
(causing weight loss), personal hygiene, and grooming. People with dementia who are in
employment may find that they are increasingly making mistakes at work.
In the later stages, basic ADLs such as dressing, eating, and walking become affected.
Specific features of subtypes of dementia:
For Alzheimer’s disease:
Early impairment of episodic memory — this may include memory loss for recent events,
repeated questioning, and difficulty learning new information.
For vascular dementia:
Stepwise increases in the severity of symptoms — subcortical ischaemic vascular dementia
may present insidiously with gait and attention problems and changes in personality.
Focal neurological signs (such as hemiparesis or visual field defects) may be present.
For dementia with Lewy bodies:
Repeated falls, syncope or transient loss of consciousness, severe sensitivity to
antipsychotics, delusions, and hallucinations may be present. Memory impairment may not
be apparent in early stages. Parkinsonian motor features (such as shuffling gait, rigidity,
bradykinesia, and loss of spontaneous movement) and autonomic dysfunction (such as
postural hypotension, difficulty in swallowing, and incontinence or constipation) may be
present.
For frontotemporal dementia (FTD):
Personality change and behavioural disturbance (such as apathy or social/sexual
disinhibition) may develop insidiously. Other cognitive functions (such as memory and
perception) may be relatively preserved.

Differential diagnosis
Conditions that can present with similar symptoms to dementia include:
Normal-age related memory changes - Normal ageing is associated with a mild decline in
cognitive function, and memory lapses are common, especially during times of physical illness or
stress.
Mild cognitive impairment (MCI) - MCI differs from dementia in that symptoms do not fulfil the
diagnostic criteria for dementia, for example only one cognitive domain may be affected or
activities of daily life may not be significantly affected. Of people with MCI, 50% will later develop
dementia.
Depression - Symptoms of depression include low mood, loss of interest, anhedonia, and self-
neglect which can be similar to those of dementia.In older people, features of depression may be
less obvious, with somatic symptoms (such as reduced appetite, fatigue, and insomnia) are more
common.
Delirium - Delirium is an acute, fluctuating syndrome of disturbed consciousness, attention,
cognition, and perception. It is a common condition in the differential diagnosis for dementia.
People with cognitive impairment are at increased risk of delirium, and the two conditions often
coexist.
Vitamin deficiency - Thiamine deficiency can lead to Wernicke encephalopathy and Korsakoff
psychosis. Symptoms include confusion, memory loss, problems with learning new information
and gait disturbances. Vitamin B12 deficiency can lead to ataxia, psychiatric abnormalities,
memory loss, and gait disturbance.
Hypothyroidism - Symptoms of hypothyroidism can include low mood, and impaired
concentration and memory.
Adverse drug effects - Many drugs, including benzodiazepines, analgesics (such as opioids,
naproxen, and ibuprofen), anticholinergics, antidepressants (such as tricyclics), antipsychotics
(such as haloperidol), anticonvulsants (especially older preparations, such as phenytoin and
(suc as a ope do ), a co u sa s (espec a y o de p epa a o s, suc as p e y o a d
phenobarbital), and corticosteroids can affect cognition.
Normal pressure hydrocephalus - Normal pressure hydrocephalus can present with symptoms of
early cognitive impairment, urinary incontinence, and gait disorder. In NPH, cerebrospinal fluid
(CSF) flow has been impeded but compensatory mechanisms have prevented raised pressure.
The lateral ventricles are prominently dilated, and these ventricles exert local pressure on certain
brain areas to give the classical clinical picture. Pressure on the frontal lobes gives the dementia
and pressure on the medial side of the motor cortex, and the pyramidal tract fibres, cause
incontinence and pyramidal leg weakness. Diagnosis is by lumbar puncture (to demonstrate a
normal CSF opening pressure) followed by head computed tomography (CT)/magnetic resonance
imaging (MRI) (showing enlarged ventricles). Treatment is with ventriculoperitoneal shunting.
Sensory deficits - Problems with vision and hearing can contribute significantly to an apparent
decline in cognitive ability.

Investigations
Patient's should be assessed using a standardised cognitive assessment tool.
To help identify reversible causes of dementia and exclude other causes of symptoms initial
investigations include:
In most cases:
Full blood count
Erythrocyte sedimentation rate (ESR)
Urea and electrolytes
Calcium
HbA1c
Liver function tests
Thyroid function tests
Serum B12 and folate levels
If clinically indicated:
A midstream urine, for example when delirium is a possibility
Chest X-ray, electrocardiogram (ECG), syphilis serology, and HIV testing
Patients with suspected dementia should be referred to a specialist memory assessment service
for further specialist assessment and management.
Specialist investigations for suspected dementia:
Should include structural imaging (MRI or CT scan) to: exclude non-dementia cerebral
pathology such as normal pressure hydrocephalus and to identify dementia subtype.
May also include:
HMPAO SPECT and FDG PET as second line investigations if diagnosis or subtype is
unclear
(FP-CIT) SPECT where a potential diagnosis of dementia with Lewy bodies (DLB) is in
doubt
Cerebrospinal fluid examination to exclude inflammatory, infective, or malignant
causes of dementia, in cases where dementia is rapidly progressive, the presentation
is unusual, or the person is younger than 55 years of age
Electroencephalography if a diagnosis of delirium, frontotemporal dementia, or
Creutzfeldt–Jakob disease is suspected, or in the assessment of associated seizure
disorder in people with dementia
Brain biopsy in highly selected people whose dementia is thought to be due to a
potentially reversible condition (such as cerebral vasculitis) that cannot be
diagnosed in another way

Management
Specialist non-pharmacological interventions for cognitive symptoms of dementia:
Structured group cognitive stimulation programs
Reminiscence therapy with discussion of past experiences
Tools such as life histories, shared memories, and familiar objects from the past
Specialist drug treatments for cognitive symptoms of dementia:
Acetylcholinesterase (AChE) inhibitors (donepezil, galantamine, and rivastigmine) - These
drugs can be used for mild to moderate Alzheimer's disease.
Memantine (a N-methyl-D-aspartic acid receptor antagonist) - Memantine is an option for
managing Alzheimer's disease for people with moderate Alzheimer's disease who are
intolerant of, or have a contraindication to, AChE inhibitors or in severe Alzheimer's disease.
Specialist drug treatment for behavioural and psychological symptoms of dementia (BPSD) or
non-cognitive symptoms of dementia and challenging behaviour:
No antipsychotics (except risperidone in very specific circumstances) are licensed for
BPSD in the UK. Specialists may prescribe an antipsychotic drug off-label in certain
circumstances (e.g. if they are severely distressed or there is an immediate risk of harm to
the person or others) following a thorough clinical assessment and discussion (including
the risks and benefits) with the carer and (if possible) the person with dementia.
Antipsychotics have potentially serious adverse effects, including increased risk of stroke
and mortality, Parkinsonism, and cognitive impairment. They should be used with caution,
at low dose, and for the shortest time possible.
In specific situations, specialists may offer AChE inhibitors for BPSD if non-
pharmacological methods and anti-psychotic drugs are inappropriate or ineffective.

You might also like